Download as pdf or txt
Download as pdf or txt
You are on page 1of 121

Strabismus (No=290)

A 30-year-old male complains of torsional diplopia since closed head trauma in a motorcycle
accident 6 months ago. On examination, there is excyclotorsion measuring 12 degrees. What
procedure is indicated:
observation
bilateral Harada-Ito surgery
bilateral superior oblique resection
bilateral inferior oblique myectomy

Your answer was CORRECT


Explanation
The most common traumatic nerve palsy following closed head injury is the fourth cranial nerve,
and bilateral involvement is common.

The following features suggest bilateral as opposed to unilateral fourth nerve palsy:
 excyclotorsion greater than 10 degrees
 V pattern
 chin down head position
 reversal of hypertropia with gaze positions
 reversal of hypertropia with head tilt: LHT on tilt left and an RHT on tilt right
 minimal hypertropia in primary position (as each palsy cancels the other).
The preferable treatment for the torsional diplopia encountered in bilateral fourth nerve pasly is
the Harado-Ito procedure. This involves displacing the anterior fibres of the superior oblique
tendon temporally.

This question came in the 2014 FRCOphth.

The inferior oblique inserts on which quadrant of the globe:


anteromedial
posteromedial
anterolateral
posterolateral

Your answer was CORRECT


Explanation
Figure: Lateral view of the globe demonstrating the insertion of the inferior oblique in the
posterior-lateral quadrant of the globe
The inferior oblique inserts on the posterolateral quadrant, mostly below the horizontal.

A 62-year-old man presents to A&E with 1-day history of diplopia. On cover test he has a
manifest right esotropia. A red glass is placed over the left eye and the patient is asked to fixate at
a distant point-light target. In the absence of suppression, and with normal retinal correspondence,
the patient should perceive the red light:
above the white light
below the white light
the lights will appear to be superimposed
to the left of the white light
to the right of the white light

Your answer was CORRECT


Explanation
Esotropia gives uncrossed diplopia on red glass testing. Thus, the left light (red) should appear to
the left of the right, white light.

An 8-year-old boy has an exotropia when looking into the distance. For near, he has an exophoria
with good binocular vision. With the addition of a +3.00DS lens for near vision, his binocular
vision is maintained.

What is the diagnosis?


simulated intermittent distance exotropia
convergence insufficiency exotropia
intermittent exotropia for near
true intermittent distance exotropia

Your answer was CORRECT


Explanation
Intermittent distance exotropia can be simulated or real. In the simulated (most common) type, the
control for near vision is due to accommodative convergence with a high AC:A ratio.

Since this patient maintains single vision for near when the drive for accommodation is removed
by +3.00DS lenses, his intermittent distance exotropia is true and not simulated.

Parents bring their 3-year-old boy for examination having noticed a squint. It has been present
throughout the day since he was 2 years old. A brief inspection of the child shows an obvious,
right, constant, moderate-angle esotropia. A cycloplegic refraction is performed and reveals
+8.50DS in both eyes.

Which of the following is TRUE?


the deviation will definitely be greater at near than at distance
there may be bilateral amblyopia
+ 3.00D lenses are likely to significantly reduce the distance deviation
the deviation at distance is likely to measure >50 prism diopters

Your answer was CORRECT


Explanation
This patient probably has refractive accommodative esotropia. Amblyopia is possible bilaterally,
given the high ametropia. The deviation may be greater at near than distance, but this is not highly
likely because patients with refractive accommodative esotropia typically have normal AC/A
ratios. In this type of esotropia, the turn is usually moderate (20 to 40 prism diopters), although it
may be greater. Lenses of +3.00D may lessen non-refractive accommodative esotropia but
typically have little effect on this type (with +3.00D lenses, there would still be a residual 5D of
hyperopia).

Which is FALSE regarding angle kappa:

can be measured with a synoptophore

if large it will cause a pseudo-esotropia


is the angle between the line of sight and the anatomical centre of the corneal-pupillary axis

is usually 5 degrees

Your answer was CORRECT


Explanation
An enlarged angle kappa, as seen with temporal displacement of the macula, will cause a pseudo-
exotropia. Other options above are true.

A large angle kappa occurs when there is temporal displacement of the macula, as with:

 ROP
 FEVR
 combined hamartoma of RPE and retina
 persistent posterior fetal vasculature
High myopia is likely to cause a negative angle kappa, which leads to pseudo-esotropia.

A patient presents with right esotropia. To estimate the deviation, the examiner chooses to use the
Krimsky method.

To do this most reliably he should:


multiply the distance of the decentered light reflex from the center of the pupil (in millimeters) by
15 to estimate the deviation in prism diopters
place prisms over the fixing eye until the corneal light reflex from the non-fixing eye appears
central
place prisms over the non-fixing eye until its light reflex appears centered
perform simultaneous prism-cover testing until there is no net movement of either eye
Your answer was CORRECT
Explanation
The Hirschberg test uses the decentration of the light reflex from the centre of the pupil to
estimate the deviation in prism dioptres (with 1mm decentration roughly equivalent to 15 prism
dioptres).

The Krimsky test is essentially the Hirschberg test, but with prisms to quantify the deviation.
Prisms can be placed over one or both eyes. However, since observation of the corneal light reflex
through a prism may be difficult, it is recommended to place the prism over the fixating eye until
the corneal light reflex from the non-fixing eye appears central.

What is the most common childhood exotropia?


consecutive exotropia
infantile exotropia
secondary exotropia
intermittent exotropia

Your answer was CORRECT


Explanation
Intermittent exotropia is the most common form of childhood exotropia. It usually presents
between 2-5 yrs of age.

This question came in the 2014 FRCOphth.

All of the following muscles have their origin in the tendinous ring EXCEPT:
lateral rectus
medial rectus
superior oblique
inferior rectus

Your answer was CORRECT


Explanation
The superior oblique originates superomedial to the optic canal. The inferior oblique originates
behind the orital margin lateral to the nasolacrimal duct. All other extraocular muscles (that is all
four recti) originate from the tendinous ring.

A 26-year-old emmetropic patient has a 6-month history of difficulty with reading. On alternate
cover testing, she is orthophoric at distance and has an exodeviation of 15 PD at near. The best
treatment option would be:
unilateral recess-resect procedure
orthoptic therapy with a base out prism or pencil push-up exercises
+ 2.00 D reading glasses
bilateral medial rectus recessions

Your answer was CORRECT


Explanation
This patient demonstrates convergence insufficiency. This condition is best treated with orthoptic
convergence training exercises, for example base out prism therapy and pencil push-up exercises
to build up convergence amplitudes. Very few strabismologists, if any, advocate surgery for this
problem.

Reading glasses would help accommodative insufficiency but not convergence insufficiency.

A patient presents with the following:

primary position: 25 PD ET
upgaze: 45 PD ET
downgaze: 5 PD ET
motility: bilateral depression with attempted adduction

Appropriate surgical intervention might include each of the following steps EXCEPT:
tenotomy of both superior oblique muscles
recession of the contralateral medial rectus muscle
upward transposition of the ipsilateral medial rectus muscle and downward transposition of
the ipsilateral lateral rectus muscle
recession of the ipsilateral medial rectus muscle

Your answer was CORRECT


Explanation
In this case, there is an A-pattern esotropia of 35 prism diopters. Overaction of the superior
obliques may be addressed with superior oblique tenotomies bilaterally. This will create an
esoshift of up to 40 prism diopters in downgaze (only). This only makes the deviation more
comitant. Recess-resect (or bimedial recession) procedures must be included. Because oblique
surgery is indicated, rectus transpositions are not.

Which is TRUE regarding consecutive exotropia?


there is usually limited adduction
over-correcting myopia can help symptoms
usually occurs after initial surgery for exotropia
more common in myopes

Your answer was CORRECT


Explanation
Consecutive exotropia usually occurs after initial surgery for esotropia. It is more common in high
hyperopes in whom refractive correction reduces the drive to converge. Decreasing the hyperopic
correction (or in myopes increasing the myopic correction) may be beneficial in decreasing the
exotropia (by driving convergence). Limitation of adduction may occur in a minority (about 30%)
of cases. Limitation of versions is unusual and suggests a slipped medial rectus or a contracted
lateral rectus may be the cause of the consecutive exo, and should be taken into account in surgical
planning.

This question appeared in the 2014 FRCOphth Part 2.

All are true of Panum's fusional space EXCEPT:

all objects are seen singly and stereoscopically

it is deeper at fixation than in the periphery


all objects stimulate non-corresponding retinal points

measures 6 seconds of arc at fixation

Your answer was CORRECT


Explanation
Panum's fusional space is:

 a zone in front and behind the horopter


 all objects stimulate non-corresponding retinal points
 all objects are seen singly and stereoscopically
 it is shallower at fixation than in the periphery
 measures 6 seconds of arc at fixation and 30-40 seconds peripherally
 objects beyond Panum's space appear double (physiological diplopia)

A 9-year-old girl is noted to have an abnormality of eye movement on a routine orthoptic school
screening visit. She has diminished abduction and markedly diminished adduction.

What is the most likely diagnosis?


Duane's syndrome
congenital fourth

infantile esotropia

Brown's syndrome

congenital sixth

Your answer was CORRECT


Explanation
Of the options provided, Duane's syndrome is the most likely to cause a combined limitation of
abduction and adduction.

Duane's retraction syndrome (DRS):


 unilateral or bilateral abnormality of horizontal gaze
 caused by co-contraction of medial and lateral recti
 retraction of the globe on attempted adduction (narrowing of palpebral apperture)
 upshooting or downshooting of the globe on adduction
 the left eye is affected more frequently than the right
 females are affected more frequently than males
 Type 1: abduction limitation greater than adduction, eso (most common)
 Type 2: adduction limitation greater than abduction, exo (least common)
 Type 3: abduction and adduction limited equally
 amblyopia in 10%

Associations with Duane's retraction syndrome:


 cataracts
 iris anomalies
 Marcus Gunn jaw winking
 microphthalmos
 crocodile tears
 Goldenhar's syndrome
 maternal thalidomide
 Klippel-Feil syndrome

This question came in the FRCS (Glasgow) October 2014 exam.

To maximize the elevation generated by the superior rectus, how must the eye be rotated from
primary position?
adducted 67 degrees

adducted 51 degrees

abducted 23 degrees
adducted 23 degrees

abducted 51 degrees

Your answer was CORRECT


Explanation
With the eye abducted 23 degrees, the superior rectus is parallel to the visual axis, and its
contraction will result in maximal elevation. For the same reason, in this position the inferior
rectus is maximized as a depressor. In primary gaze, contraction of the superior rectus not only
elevates the eye but also intorts and adducts the eye. (The inferior rectus will extort and adduct the
eye, as well as depress it, in primary gaze.)

Some helpful tips on ocular muscles:


 the primary action of the obliques is intorsion/extorsion
 the primary action of the recti is in the direction of their insertion (medial adduct, superior
elevates, etc)
 horizontal recti in primary position have no vertical/torsional action
 horizontal recti in upgaze weak elevators; in downgaze, weak depressors
 vertical recti (sup and inf) always ADDuct as a secondary action in primary position
 obliques always ABDuct as a secondary action in primary position
 superior muscles INtort
 inferior muscles EXtort
 39 degrees abduction isolates primary action of the obliques (intort/extort)
 51 degrees adduction isolates secondary action of the obliques (elevate/depress and
abduction)
 23 degrees abduction isolates primary muscle of sup/inf recti (elevate/depress)
 67 degrees adduction isolates secondary action of sup/inf recti (intort/extort)

Conditions that cause a chin-up position include all of the following EXCEPT:
bilateral SO palsy
double elevator palsy
A pattern ET
V pattern XT

Your answer was CORRECT


Explanation
Patients with bilateral SO palsy typically have IO over-action with a V-pattern ET and chin down
head position.

A child with strabismus is asked to fixate on a penlight held by the examiner. The examiner notes
that the corneal reflex in the right eye is central, whereas that in the left eye is displaced
approximately 3 mm temporal to the center of the pupil.

Using Hirschberg's method for estimating the angle of strabismus, the examiner concludes that the
patient has a:
45-prism-diopter exotropia
45-degree esotropia
45-prism-diopter esotropia
30-degree exotropia
30-prism-diopter esotropia

Your answer was CORRECT


Explanation
Each millimeter of decentration is roughly 7 degrees of deviation. Each degree is approximately 2
prism diopters. Thus, each millimeter is approximately 15 prism diopters. Three millimeters is 21-
plus degrees, or 45 prism diopters. Because the reflex is displaced temporally, the eye must be
deviated inward (esotropia).
A 7-year-old boy presents via his opticians with visual acuity of 6/6 right eye; 6/15 left eye.
Motility is full, and there is no apparent tropia on cover-uncover testing. The child has
stereoacuity of 120 seconds of arc. Distance Worth four-dot testing reveals fusion. Convergence
and divergence amplitudes are normal at distance. There is no movement detected when a 4-prism
dioptre is placed over the left eye.

What additional finding is MOST likely:


limitation of elevation most marked in adduction
retraction of the globe on adduction
high axial myopia bilaterally
hypermetropic anisometropia

Your answer was CORRECT


Explanation
The clinical features of mild amblyopia, mildly impaired stereopsis, peripheral fusion, eccentric
fixation with a central suppression scotoma are typical of a microtropia (the tropia is either too
small to detect or does not exist because of parafoveal fixation and anomalous retinal
correspondence). Microtropia is most common in the setting of anisometropia, usually
hypermetropic anisometropia. Microtropia can also occur after (almost) successful strabismus
surgery where the child controls to a microtropia.

Which is FALSE regarding accommodative esotropia:

by definition patients have a high AC/A ratio

patients rarely develop diplopia

less than half of patients are amblyopic

usually intermittent at onset before becoming constant

Your answer was CORRECT


Explanation
Accommodative esotropia is:

 acquired
 develops as the child begins to accommodate around the age of 1 to 2 yrs
 patients usually suppress the deviated eye - therefore no diplopia
 because the deviation is acquired, many patients alternate fixation
 less than half will have amblyopia
 usually hyperopic with greater than 3.00 D of hyperopia
 20% have a high AC/A ratio and will benefit from bifocals

Which is TRUE of dissociated vertical deviation?

dampens when the eye is under cover


is an example of a condition that disobeys Sherington's law

occurs in approximately 80% of infantile esotropias by 3 years

usually unilateral

Your answer was CORRECT


Explanation
DVD is characterised by:

 updrift with excyclotorsion of the eye


 exacerbated under cover and by inattention
 occurs in 60% to 80% of patients with congenital esotropia
 usually bilateral and asymmetric
 aetiology unknown
 possibly due to early disruption of binocular development
 high-grade stereopsis and bifoveal fixation are not seen
 disobeys Hering's law: eye drifts upwards under cover, but when the cover is removed
and the eye comes back down, it is NOT associated with a conjugate downward
movement of the other eye

Which of the following constitutes a violation of Sherrington's law?


Duane's syndrome
inhibitional palsy of the contralateral antagonist
DVD
Brown's syndrome

Your answer was CORRECT


Explanation
In Duane's syndrome, innervational impulses to the medial rectus are not associated with
decreased innervation of the ipsilateral lateral rectus muscles, as Sherrington's law demands. This
is because of anomalous innervation of the lateral rectus, in this case from the oculomotor (III)
nerve, which leads to coinnervation of the horizontal rectus muscles, with globe retraction on
adduction.

A rectus muscle transposition would be LEAST appropriate in which of the following scenarios?
non-resolving sixth nerve palsy
dissociated vertical deviation
double elevator palsy
non-resolving third nerve palsy

Your answer was CORRECT


Explanation
DVD is frequently treated by anteriorisation of the inferior oblique tendon to Marshall Parks point
3mm lateral and 1 mm behind the lateral border of the inferior rectus; it is not treated by
horizontal rectus transposition.

When the force-generating capability of a muscle is permanently and significantly depressed (e.g.
sixth nerve palsy, third nerve palsy and double-elevator palsy), resection techniques will offer
little improvement. Here, transposition of neighboring, healthy rectus muscles (e.g. Jensen's
procedure for a sixth) is often helpful.

All are true of Panum's space EXCEPT:


the Panum's area is wider in the periphery
Panum's area is a zone in front of and behind the horopter
stereopsis exists in this space
all objects stimulate corresponding retinal points

Your answer was CORRECT


Explanation
Objects on the horopter line stimulate corresponding retinal points. In the Panum's area, non-
corresponding points are stimulated; this disparity is used to produce the perception of depth
(stereopsis).

A 7-year-old boy presents with an exotropia. His measurements are as follows:

Visual acuity: 6/6 both eyes


PCT distance (primary position): 30 PD XT
PCT distance (right gaze): 20 PD XT
PCT distance (left gaze): 40 PD XT
PCT (near): 15 PD XT in all directions
Fixation appears to alternate.

The boys parents strongly desire some form of correction. What is the most appropriate
management:
bilateral lateral rectus recessions with greater recession on the right eye
base-in prism
addition of +2.00D to his current distance refraction
bilateral lateral rectus recessions with greater recession on the left eye
bilateral lateral rectus recessions, equal on each side

Your answer was CORRECT


Explanation
Base-in prism is not typically recommended for distance XT, because it reduces the convergence
fusional amplitudes which are controlling or counteracting the squint, worsening the situation
without glasses. An increased myopic (not hyperopic) prescription could be used to stimulate
accommodative convergence, though it is unlikely to solve the problem. Because the parents are
determined to correct the cosmetic problem, and because the deviation is greater in left gaze,
weakening of both lateral recti, more so on the left, should be undertaken.

All are true of convergence insufficiency EXCEPT:


it presents in school age children rather than infants
pencil push-ups are recommended over glasses
reduced near point of accommodation
it can be a post-viral phenomenon

Your answer was CORRECT


Explanation
Convergence insufficiency refers to a reduced near point of convergence (not a reduced near point
of accommodation). The treatment of choice is orthoptic exercises. It can occasionally present
with a degree of accommodative insufficiency also, in which case glasses can be prescribed. It
may be idiopathic or a post-viral phenomenon

Which of the following arises from the annulus of Zinn?


superior oblique
levator palpebrae superioris
superior rectus
inferior oblique

Your answer was CORRECT


Explanation

The superior oblique and the levator muscles each arise posteriorly, above the annulus. The
inferior oblique arises from the anteromedial orbital floor. The annulus gives rise to the four rectus
muscles.
A 4-year-old boy with Down's syndrome presents with his mother, who complains of a constant
esotropia. On examination, the corneal reflex is central in the right eye, while the reflex lies
midway between the pupil edge and the temporal limbus in the left eye.

What is the approximate angle of deviation?


20 prism dioptres
80 prism dioptres
30 prism dioptres
60 prism dioptres
50 prism dioptres

Your answer was CORRECT


Explanation
This is the Hirschberg test where each 1 mm displacement represents 15 PD

Displacement to pupillary margin = 2mm = 30 PD


Displacement to mid-iris = 4 mm = 60PD
Displacement to limbus = 5.5mm = 80PD

This question came in the FRCS (Glasgow) October 2014 exam.

A 72-year-old diabetic has a unilateral total sixth cranial nerve palsy for the past 2 years.

What is the most appropriate surgical management?


Inverse Knapp procedure
maximum medial rectus resection and lateral rectus recession
Harada-Ito procedure
Hummelsheim procedure
Knapp procedure

Your answer was INCORRECT


Explanation
Figure: Hummelsheim procedure

In a total sixth nerve palsy, the eso-deviation cannot be entirely overcome by a maximal medial
rectus-lateral rectus recess/resect procedure. Therefore alternative methods are often employed.

The Hummelscheim procedure involves disinserting the lateral halves of the superior and inferior
recti and reattaching them above and below the lateral rectus, while the medial rectus is recessed.

Another option here is the Jensen procedure where the superior, inferior and lateral recti are split
into halves about 15mm from their insertion without disinsertion. The halves are tied loosely
together in pairs, with the main advantage of this method being the reduced risk of anterior
segment ischaemia from disinsertion of the recti.

A patient presents has the following:

primary position: XT 15 PD
downgaze: XT 5 PD
upgaze: XT 30 PD
motility: significant elevation of each eye with adduction

Appropriate surgical intervention might include all of the following steps EXCEPT:
recession of the ipsilateral lateral rectus muscles
bilateral inferior oblique myectomies
recession of the contralateral lateral rectus muscle
upward transposition of the lateral rectus and downward transposition of the medial rectus
muscles ipsilaterally

Your answer was CORRECT


Explanation
This is a case of 15 PD V-pattern, which is clinically significant. In alphabet patterns, one should
search for overaction or underaction of the obliques. If they are present, then these should be
addressed surgically to correct the alphabet pattern. In this case there is significant overaction of
the inferior obliques. Bilateral inferior oblique myectomies will lessen the V pattern by 15 to 25
prism dioptres. The horizontal component can then be addressed by bilateral LR recessions or a
unilateral recess/resect.

If oblique dysfunction is absent, then alphabet patterns should be addressed by vertical


transposition of the horizontal recti using the pnemonic:

MALE: Medial rectus toward the Apex and Lateral rectus toward the Empty space. This holds for
both A patterns and V patterns.

If a Maddox rod is in front of the right eye and a patient sees the red line above the white dot this
indicates:

right exophoria

right hypophoria
horizontal orthophoria

right hyperphoria

Your answer was CORRECT


Explanation
Figure: The patients view of a Maddox rod before the right eye.

1. Horizontal orthophoria
2. Exophoria (the patient has crossed diplopia and this indicates an exophoria (X'ed diplopia =
eXo)
3. Esophoria
4. Vertical orthophoria
5. Right hyperphoria
6. Right hypophoria (=left hyperphoria)

Maddox rod test interpretation:

 Line above point: covered eye is hypo-deviated


 Line below point: covered eye is hyper-deviated
 Line temporal: covered eye is eso-deviated
 Line nasal: covered eye is exo-deviated

The actions of the medial rectus muscle with the eye in primary position are:
adduction

adduction, depression, intorsion

adduction and intorsion

adduction and extorsion

adduction, elevation, intorsion

Your answer was CORRECT


Explanation
The horizontal rectus muscles have no torsional or vertical action in primary gaze. In upgaze, they
are elevators (weak); in downgaze, they are depressors (weak).

Some helpful rules to try and remember the muscle actions:


 the primary action of the obliques is intorsion/extorsion
 the primary action of the recti is in the direction of their insertion (medial adduct, superior
elevates, etc)
 horizontal recti in primary position have no vertical/torsional action
 horizontal recti in upgaze weak elevators; in downgaze, weak depressors
 vertical recti (sup and inf) always ADDuct as a secondary action in primary position
 obliques always ABDuct as a tertiary action in primary position
 superior muscles INtort
 inferior muscles EXtort
 39 degrees abduction isolates primary action of the obliques (intort/extort)
 51 degrees adduction isolates secondary action of the obliques (elevate/depress and
abduction)
 23 degrees abduction isolates primary muscle of sup/inf recti (elevate/depress)
 67 degrees adduction isolates secondary action of sup/inf recti (intort/extort)
All of the following correctly match a muscle with its synergist and antagonist EXCEPT:
medial rectus: synergist, superior rectus; antagonist, lateral rectus
inferior oblique: synergist, superior rectus; antagonist, superior oblique
inferior rectus: synergist, superior oblique; antagonist, superior rectus
superior rectus: synergist, superior oblique; antagonist, inferior rectus
lateral rectus: synergist, superior oblique; antagonist, medial rectus

Your answer was CORRECT


Explanation
The main synergist of the superior rectus is the inferior oblique (both elevate the globe)

To maximize the elevation generated by the inferior oblique, how must the eye be rotated?
adducted 51 degrees
abducted 51 degrees

adducted 67 degrees

abducted 23 degrees

adducted 23 degrees

Your answer was CORRECT


Explanation
With the eye adducted 51 degrees, the tendon of the inferior oblique is parallel to the visual axis,
and its contraction will result in maximal elevation. For the same reason, in this position the
superior oblique will be maximized as a depressor. In primary gaze, contraction of the inferior
oblique is primarily extorsion, but also elevation and abduction. (The superior oblique will intort
and abduct the eye, as well as depress it, in primary gaze.)

Some helpful tips on ocular muscles:


 the primary action of the obliques is intorsion/extorsion
 the primary action of the recti is in the direction of their insertion (medial adduct, superior
elevates, etc)
 horizontal recti in primary position have no vertical/torsional action
 horizontal recti in upgaze weak elevators; in downgaze, weak depressors
 vertical recti (sup and inf) always ADDuct as a secondary action in primary position
 obliques always ABDuct as a secondary action in primary position
 superior muscles INtort
 inferior muscles EXtort
 39 degrees abduction isolates primary action of the obliques (intort/extort)
 51 degrees adduction isolates secondary action of the obliques (elevate/depress and
abduction)
 23 degrees abduction isolates primary muscle of sup/inf recti (elevate/depress)
 67 degrees adduction isolates secondary action of sup/inf recti (intort/extort)

A 5-year-old boy presents with an exodeviation of 30 PD at distance and 10 PD at near on


alternate cover testing. A patch test is performed on this patient and he now measures 30 PD at
distance and 15 PD at near. The patient then undergoes a bilateral lateral rectus recession. One
week later, he measures 15 PD consecutive esotropia. He is observed and at 3 weeks
postoperatively, the child is still 15 PD esotropic and symptomatic.

What is the next most appropriate step?


prescribe enough base out prism to fully neutralize the esotropia
prescribe a miotic
operate for 15 PD esotropia
prescribe enough base out prism to alleviate the diplopia but leave a small residual
esophoria

Your answer was CORRECT


Explanation
Prescribe just enough prism to alleviate the diplopia but leave a residual esophoria to encourage
divergence. If the consecutive esotropia is present only at near, one can consider a bifocal add,
miotics, or even base out prism. If the esotropia persists after 8 weeks, consider reoperation
(usually a bilateral medial rectus recession). If the patient demonstrates lateral incomitance or
significant limitation of abduction, a slipped muscle is a possibility.

All of the findings below are characteristic of a left Brown's syndrome EXCEPT:

limited left elevation in adduction

downshoot of left eye in adduction

normal left elevation in abduction

left superior oblique overaction

abnormal head posture with chin lift and left head tilt

Your answer was CORRECT


Explanation
In Brown's syndrome there is absence of ipsilateral superior oblique overaction, which helps to
distinguish Brown's syndrome from a paresis of the inferior oblique. Another feature which
distinguishes Brown's from IO palsy is the forced duction test, which is positive in Brown's but
negative in IO palsy.

Brown's syndrome is characterised by:

 congenital or acquired aetiology


 inelastic superior oblique muscle tendon complex
 restriction of passive or active elevation in adduction (including positive forced duction)
 downshoot of the eye on adduction (but no superior oblique overaction)
 no superior oblique overaction
 V-pattern is common
 rarely the eye may become hypotropic
 rarely an abnormal head posture occurs with a tilt toward the affected side

A child with congenital nystagmus has a null zone in right gaze and severe left head turn. Surgical
intervention for the patient might include:

bilateral lateral rectus recession


bilateral medial rectus recession
left MR resect/LR recession and right MR recess/LR resect
right MR resect/LR recession and left MR recess/LR resect

Your answer was INCORRECT


Explanation
The goal of muscle surgery in congenital nystagmus is to move the null zone to primary position.
This is done by shifting the resting position out of primary position toward the direction opposite
to the null zone. Then, to obtain primary gaze, the child will need to innervate ocular muscles as if
gaze were toward the null zone.

Signs of ischaemia following excessive rectus muscle surgery include all EXCEPT:
anterior chamber reaction
hypotony
corneal oedema
retinal neovascularisation

Your answer was CORRECT


Explanation
The ischaemia induced by surgery on three or more muscles is entirely anterior

7-year-old boy is found to have an ET which measures 20 prism diopters in upgaze, 30 PD in


primary gaze, and 45 PD in downgaze. There is significant inferior oblique overaction (+2 to +3).

The best surgical treatment is:

bilateral LR resection with upward transposition

bilateral MR recession with upward transposition

bilateral MR recession and IO anteriorization

bilateral LR recession with IO anteriorization


Your answer was CORRECT
Explanation
An alphabet pattern with significant oblique overaction should be corrected by oblique weakening.
In the absence of oblique overaction, transposition of the recti should be undertaken using the
pnemonic: MALES

 Medial recti to apex


 Lateral recti to ends

A child is suspected of having right monofixation syndrome. Which single test would be most
helpful to confirm your suspicions:
afterimage testing
fogging refraction
Bagolini glass testing
four-prism-diopter base-out test
Lancaster red-green test

Your answer was CORRECT


Explanation
A weak base-out prism will shift a fixation target nasally. If placed over the microtropic eye, the
shift will not move the target out of the suppression scotoma, and no re-fixation movement is
generated. When placed contralaterally, both eyes will shift away from the base of the prism. No
fusional convergence will be seen contralaterally because of the central scotoma. In normal eyes,
the contralateral eye converges to fuse the fixation images.

For a strabismus to be appropriately termed congenital, the disturbance must be documented:


within the first 4 weeks of life
at birth
within the first 3 months of life
within the first 6 months of life

Your answer was CORRECT


Explanation
Congenital = detected in the first 6 months

All are recognised associations with Duane's syndrome EXCEPT:


keratoconus

short neck with low hairline


persistent hyaloid artery

anisocoria

epibulbar dermoid

Your answer was CORRECT


Explanation
Keratoconus is not known to be associated with Duane's syndrome.

Duane's retraction syndrome (DRS):


 unilateral or bilateral abnormality of horizontal gaze
 caused by co-contraction of medial and lateral recti
 retraction of the globe on attempted adduction
 upshooting or downshooting of the globe on adduction
 the left eye is affected more frequently than the right
 females are affected more frequently than males
 Type 1: abduction limitation greater than adduction, eso (most common)
 Type 2: adduction limitation greater than abduction, exo (least common)
 Type 3: abduction and adduction limited equally
 amblyopia in 10%
Ocular associations with Duane's retraction syndrome:
 cataracts
 iris anomalies including anisocoria
 Marcus Gunn jaw winking
 crocodile tears
 ptosis
 persistent hyaloid artery
 myelinated nerve fibres
 nystagmus
 microphthalmos
 epibulbar dermoids (Goldenhar's syndrome)
Systemic associations:
 maternal thalidomide
 Klippel-Feil syndrome (neck and spine abnormalities)
 Goldenar's syndrome
 deafness
 cafe-au-lait spots

All are recognised associations with Duane's syndrome EXCEPT:


keratoconus

short neck with low hairline

persistent hyaloid artery

anisocoria
epibulbar dermoid

Your answer was CORRECT


Explanation
Keratoconus is not known to be associated with Duane's syndrome.

Duane's retraction syndrome (DRS):


 unilateral or bilateral abnormality of horizontal gaze
 caused by co-contraction of medial and lateral recti
 retraction of the globe on attempted adduction
 upshooting or downshooting of the globe on adduction
 the left eye is affected more frequently than the right
 females are affected more frequently than males
 Type 1: abduction limitation greater than adduction, eso (most common)
 Type 2: adduction limitation greater than abduction, exo (least common)
 Type 3: abduction and adduction limited equally
 amblyopia in 10%
Ocular associations with Duane's retraction syndrome:
 cataracts
 iris anomalies including anisocoria
 Marcus Gunn jaw winking
 crocodile tears
 ptosis
 persistent hyaloid artery
 myelinated nerve fibres
 nystagmus
 microphthalmos
 epibulbar dermoids (Goldenhar's syndrome)
Systemic associations:
 maternal thalidomide
 Klippel-Feil syndrome (neck and spine abnormalities)
 Goldenar's syndrome
 deafness
 cafe-au-lait spots

Which of the following outcomes regarding lid position is NOT consistent with the muscle
surgery described?
narrowing of palpebral fissure with superior rectus resection
narrowing of palpebral fissure with inferior rectus recession
no change of the palpebral fissure with inferior oblique recession
no change of the palpebral fissure with superior oblique tenotomy

Your answer was CORRECT


Explanation
Because the superior rectus is loosely attached to the levator complex, recession will pull the
levator back and widen the fissure, whereas resection will pull the levator forward and narrow the
fissure. The same relationships apply for the inferior rectus because its sheath gives rise to the
capsulopalpebral fascia. Operation on the superior and inferior obliques should not affect the
palpebral fissure.

An adjustable suture technique would be advisable in each of the following scenarios EXCEPT?
strabismus associated with Graves' disease
disassociated vertical divergence
infantile esotropia with manifest-latent nystagmus
muscle transposition surgery in A- or V-pattern horizontal strabismus

Your answer was CORRECT


Explanation
Adjustable squint surgery is not advisable for paediatric cases, as children do not tolerate the
adjustment process.

For the other options provided above, adjustable squint surgery is particularly important, since the
angle of deviation is difficult to quantify or the effects of muscle surgery are hard to predict

Which statement regarding paediatric ophthalmology evaluation is FALSE:


a Cardiff card is a reasonable option for testing the acuity of a 12-month-old
a child with eccentric fixation will usually have vision of 6/60 or worse
the cover-uncover test is used to identify a tropia
temporal displacement of the light reflex that does not shift during cover-uncover or
alternate cover testing represents a positive angle kappa

Your answer was CORRECT


Explanation
Positive angle kappa represents a slight temporal position of the fovea relative to the optical axis.
This causes a slight temporal rotation of the globe to keep the image on the fovea. This, in turn,
causes the corneal light reflex to be displaced nasally. The deviated light reflex remains stable
with cover testing as the fovea never spontaneously moves relative to the optical axis.

A patient with right esotropia measuring 50PD for near and 30PD for distance would like to have
squint surgery. There is no previous history of surgical correction.

On the basis of the clinical information provided above, which procedure is MOST likely to be
appropriate:
bilateral medial rectus recession
right medial rectus recession with lateral rectus resection
bilateral lateral rectus resection
left medial rectus recession with lateral rectus resection

Your answer was CORRECT


Explanation
For deviation greater at near, surgery is recommended on the medial recti, while for deviations
greater in the distance, surgery is recommended on the lateral recti.

A patient takes the Worth 4-dot test wearing the green lens in front the right eye and the red lens
in front of the left eye. She reports seeing 5 lights, 2 red and 3 green.

What is the most appropriate conclusion?

she has normal BSV

she has left suppression

she suffers from diplopia

she has harmonious abnormal retinal correspondence

she has functional visual impairment

Your answer was CORRECT


Explanation
In a Worth 4-dot test, there are 4 lights: 1 red, 2 green and 1 white.

If the patient wears red-green glasses with the green lens in front of the right eye, and the red lens
in front of the left eye, then the following outcomes can be interpreted:

 4 lights seen and patient is orthophoric = normal BSV


 4 lights seen and patient has manifest squint = harmonious ARC
 2 red lights seen = right suppression
 3 green lights seen = left suppression
 5 lights (2 red and 3 green) = diplopia

Note: typically the red lens is placed in front of the right eye and the green in front of the left eye,
but the test can be performed either way around, and it is important to be able to interpret the
results either way.

Which rectus muscle receives only one ciliary artery:


medial rectus
lateral rectus
superior rectus
inferior rectus

Your answer was CORRECT


Explanation
All rectus muscles receive two ciliary arteries except the lateral rectus which receives just one.

During a routine examination, the cover-uncover test reveals an outward fixation shift of either
eye as the cover is placed over the contralateral eye. The alternate-cover test reveals no shift as the
cover is moved back and forth.

The correct description of this patient's motility status would be:


orthophoric, orthotropic
orthotropic, exophoric
orthotropic, esophoric
orthophoric, esotropic
this set of findings is not possible

Your answer was CORRECT


Explanation
In the setting of an esotropia (detected by cover-uncover test), the alternate-cover test must
disclose some strabismus because it detects both tropia and phoria. The cover-uncover test detects
only tropia.

All are recognised side-effects of Botox injection for strabismus EXCEPT:


vertical and torsional squints
scleral perforation
diplopia
ptosis
miosis

Your answer was CORRECT


Explanation
Botox may cause mydriasis (not miosis), by its effect on ACh release at the parasympathetic
neuromuscular junction. Other options are true.

Botulinum toxin is most likely to be effective in the management of which of the following
conditions?
acute lateral rectus palsy
congenital nystagmus
large angle congenital esotropia
dissociated vertical deviation
intermittent distance exotropia

Your answer was CORRECT


Explanation
Botox is a temporary measure, and can be useful in the setting of an acute lateral rectus palsy to
reduce symptomatic diplopia while the paretic muscle recovers function.

Botox has limited value in the treatment of congenital strabismus, nystagmus and DVD (beyond
pre-operative diplopia testing), as conventional muscle surgery remains the treatment of choice in
these conditions. Botox has been suggested as an alternative to surgery in small to medium angle
esotropia; although surgery remains the most popular treatment option even in these cases.
1

Reference: 1. de Alba Campomanes AG et al. J AAPOS. 2010;14(2):111-116.

Which is TRUE regarding strabismus associated with Graves disease?


prisms are frequently a well-tolerated, permanent solution for diplopia
strabismus surgery should be performed before eyelid surgery
surgical management of diplopia is best achieved by muscle resections with adjustable sutures
strabismus surgery should be performed before orbital radiation

Your answer was CORRECT


Explanation
The deviations of Graves' disease are classically noncomitant and not easily addressed with
prisms. Adjustable sutures are very helpful in this condition (which may respond unpredictably to
muscle surgery). Resection techniques, however, are generally avoided because Graves' disease
generally causes considerable restriction. Resection would tend to exacerbate this and leave an eye
nearly frozen if overdone.

Muscle surgery, particularly vertical muscle surgery often indicated in Graves' disease, can affect
the position of the eyelids. As a result, it is generally wise to perform any strabismus surgery
before eyelid repositioning.

However, strabismus surgery should be performed after treatments for orbital disease have been
completed including orbital decompression and radiation, as these treatments are more urgent, and
they can alter the strabismic effects of TED and change decisions for surgery.

Each millimeter of recession of a vertical rectus muscle will result in approximately what vertical
correction:

8 prism diopters of vertical correction

5 prism dioptres

1 prism diptre

3 prism dioptres

Your answer was CORRECT


Explanation
 Vertical surgery: 1mm = 3 prism diopters correction
 Horizontal surgery:
o 1mm recession = 2 prism dioptres
o 1mm resection = 4 prism dioptres
Note: medial rectus recession more effective than lateral rectus recession

Which is NOT an indication for strabismus surgery in intermittent distance exotropia?


an abnormal head posture
increased frequency of breakdown to diplopia
decreasing stereopsis
an increasing angle on prism cover test

Your answer was CORRECT


Explanation
An increasing angle on prism cover test is not in itself an indication for surgery if the patient is
able to comfortably control the angle.

An increased frequency of breakdown to diplopia, loss of stereopsis and abnormal head postures
are all indications for surgery.

All of the following are typically associated with a V-pattern EXCEPT:

fourth nerve palsy

congenital esotropia

Brown's syndrome

inferior rectus underaction

Your answer was CORRECT


Explanation
V-pattern is caused by:

 inferior oblique overaction (e.g. fourth nerve or childhood esotropia)


 superior oblique underaction
 superior rectus underaction
 Browns syndrome
 craniofacial anomalies
A-pattern is caused by:
 superior oblique overaction
 inferior oblique underaction
 inferior rectus underaction
Which of the following is MOST likely to be helpful in the treatment of intermittent exotropia:
phospholine iodide
base-in prism
minus lenses
bilateral lateral rectus resections

Your answer was CORRECT


Explanation
Minus lenses stimulate accommodation and may provide the extra convergence needed for fusion.
Base-in prisms may permit fusion but are not universally advocated because they may weaken
fusional convergence and convert an intermittent XT to a constant deviation.

Phospholine (a long-acting cholinesterase inhibitor which causes miosis and accommodation


without convergence) plays a role in accommodative esotropia, not exotropia. Lateral rectus
resections strengthen the lateral rectus and will exacerbate exotropia.

a patient diagnosed with persistent, intractable near-reflex spasm, which of the following is MOST
likely to be appropriate?
base-out prisms

atropine
pilocarpine

bifocals

base-in prisms

Your answer was CORRECT


Explanation
Near reflex spasm is characterised by:
 female preponderance
 over-exuberant near reflex convergence and accommodation
 symptoms: diplopia and blurred vision
 signs: esotropia for near, pseudo-myopia, miosis
 Rx: reassurance, if persistent consider atropine with full reading prescription

Which statement regarding diplopia is TRUE?

1. Diplopia occurs when each of the two foveae of a single patient contains a distinct retinal image

2. If a patient with manifest strabismus does not complain of diplopia, then there must be
suppression
1 and 2
neither 1 nor 2
2 only
1 only

Your answer was CORRECT


Explanation
Diplopia, or double images of a single object, exists when the two images of the object fall on
non-corresponding areas of the retina. When two different objects are simultaneously imaged on
the foveas and perceived, visual confusion exists.

Although suppression is the most common explanation for absence of diplopia with manifest
strabismus, marked organic visual loss in the deviating eye will prevent diplopia, requiring no
central suppression. In large-angle esotropia, the nasal bridge may occlude the deviating eye and
prevent diplopia. Anomalous retinal correspondence also may prevent diplopia despite an obvious
tropia.

A 6-year-old girl presents from the orthoptic screening program. Visual acuity is 6/6 right eye;
6/15 left eye. Motility is full, and there is no apparent tropia on cover-uncover testing. The child
has stereoacuity of 120 seconds of arc. Distance Worth four-dot testing reveals fusion.
Convergence and divergence amplitudes are normal at distance.

The most likely diagnosis is:


Duane's syndrome
central fixation with anomalous retinal correspondence
factitious visual loss
cyclic esotropia
monofixation syndrome

Your answer was CORRECT


Explanation
The clinical features of mild amblyopia, mildly impaired stereopsis, peripheral fusion, eccentric
fixation with a central suppression scotoma are typical of a microtropia (the tropia is either too
small to detect or does not exist because of parafoveal fixation and anomalous retinal
correspondence).

A patient has a V-pattern esotropia with discrepancy in measurements between upgaze and
downgaze of 15PD. There is minimal inferior oblique overaction on motility testing, which is
otherwise normal.

On the information provided, which of the following is the most appropriate surgical
management?

LR resection with downard transposition, MR recession with upward transposition

bilateral MR recession with downward transposition

bilateral MR recession with upward transposition


LR resection with upward transposition, MR recession with downward transposition

inferior oblique weakening

Your answer was CORRECT


Explanation
For A- and V-pattern corrections, if there is significant oblique overaction, then this should be
tackled by an appropriate weakening procedure.

If there is no significant oblique overaction, then upward or downward transposition of the recti
should be undertaken.

The direction of transposition of recti: (pnemonic: MALE)

 Medial recti to Apex


 Lateral recti to ends
In general a bilateral recess procedure with transposition gives a better effect for A- and V-pattern
corrections than a unilateral recess/resect procedure. Thus the procedure of choice in this case is
bilateral MR recessions with downward transpositions.

The extraocular muscle with the shortest length of tendon is the:


superior rectus
levator palpebrae superioris
inferior oblique
superior oblique
inferior rectus

Your answer was CORRECT


Explanation
The inferior oblique consists of muscle virtually all the way to its insertion over the posterior
sclera.

When used with prisms, which of the following tests is most appropriate for quantification of a
tropia only, with no contribution from a phoria?
cover-uncover test
double Maddox rod testing
Maddox rod testing
simultaneous prism-cover test
alternate-cover test

Your answer was CORRECT


Explanation
In the simultaneous prism-cover test, as the fixing eye is covered, a prism is simultaneously placed
over the deviated eye. When there is no re-fixation shift, the prism has neutralized the tropia. The
alternate-cover test with prisms will neutralize total phoria plus tropia. The cover-uncover test
with prisms will neutralize tropia plus part of the phoria.

A 18-month-old child attends clinic with her parents who report a 6 month-history of squint.
Examination findings are as follows:

Good fixation either eye


PCT: 40PD ET near and distance
Cycloplegic refraction: + 2.00D in both eyes

What is the MOST appropriate next step in the management of this child:
bilateral medial rectus recession of 5.5 mm in both eyes
bilateral lateral rectus recession of 8 mm in both eyes
prescription of + 2.00D in both eyes with + 3.50D add in both eyes
prescription of + 2.00D glasses in both eyes
alternate patching throughout the day

Your answer was CORRECT


Explanation
Although this probably represents a case of basic esotropia, there may be an accommodative
component. There is no way to rule this out without a trial of hyperopic correction. Bifocals might
be added later if a significant residual esodeviation remains at near with distance correction. Early
surgery might be advocated to maximize the retention of fusion, but only the angle of deviation
that remains after refractive correction should be addressed. Close follow-up, as always, is critical.

Clinical features of a childhood esotropia that are predictive of the need for future surgical
intervention include all of the following EXCEPT:

low hyperopia

large-angle esotropia over 50 PD

the presence of overaction of the inferior obliques

age of onset between 2 and 3 years

Your answer was CORRECT


Explanation
Age of onset between 2 and 3 years makes an accommodative mechanism more likely, with better
prognosis for refractive correction. The other findings are consistent with a large-angle, congenital
esotropia, which usually requires surgical correction.

Congenital esotropia is characterised by:

 large angle esotropia, usually greater than 30 PD


 onset usually during the first few months, by definition by 6 months of age
 cross-fixation may be present
 latent nystagmus
 dissociated vertical deviation (in up to 60%-70%)
 inferior oblique overaction with V-pattern esotropia (in up to 60%-70%)
 mild hyperopia, + 1.00 to + 2.00 D

Which one of the following best illustrates an EXCEPTION to Sherrington's law?

convergence

Duane's type I
alternating esotropia

dissociated vertical deviation

Your answer was CORRECT


Explanation
Sherrington's law states that when one extraocular muscle is stimulated, the ipsilateral antagonist
is inhibited. In Duane's type I, the lateral rectus muscle is innervated by part of the medial rectus
subdivision of the third nerve. This may be a result of a congenital agenesis of the abducens
nucleus, which has been demonstrated pathologically. As a result of this aberrant innervation
when the medial rectus is stimulated to contract, the lateral rectus also receives stimulatory
impulses, thus violating Sherrington's law.

Hering's law states that when an extraocular muscle receives stimulation, its yoke muscle (the
prime mover in the contralateral eye in the same field of gaze) receives equal innervation. An
exception to Hering's law is DVD. In DVD, one eye elevates, extorts, and abducts without any
innervation to the contralateral eye.

Which is FALSE regarding stereoacuity:


it is greater at the centre of vision than the mid-periphery
only horizontal disparity in images will elicit stereovision
it increases as the horopter is approached and is maximal on the horopter plane
it increases as an object comes nearer

Your answer was CORRECT


Explanation
Stereoacuity increases as the horopter is approached but it is zero on the horopter plane, since
corresponding retinal points are stimulated on the horopter. Other statements are true.

Normal stereoacuity:
 Centrally: 20-40 seconds of arc
 Peripherally: 200 seconds of arc
 Maximal at 0.25 degrees off dead-centre of the fovea
 Minimal beyond 15 degrees of eccentricity

A 4-year-old boy measures XT 30 at distance and X(T)15 at near fixation. How would this
deviation be characterised?

basic exotropia

true divergence excess

pseudo-divergence excess

cannot determine from the information provided

Your answer was CORRECT


Explanation
This is a common pattern of exodeviation and requires further evaluation. Exotropia that is greater
in the distance could represent either pseudo divergence excess or true divergence excess. One test
that could be helpful is the 30-minute patch test. With this dissociating patch on for 30 minutes,
the near deviation often increases and approximates the distance deviation. This deviation would
then be classified as a pseudo-divergence excess. Using a + 3.00 D lens may also prove helpful to
look for high AC/A ratio patients. This classification is important both for determining how far to
recess the recti muscles and for prognosis.

A patient with constant exotropia has a deviation of 35 prism diopters at distance and 15 prism
diopters at near. After wearing a patch over his right eye for 1 hour, the deviations are remeasured
and found to be 35 prism diopters at distance and 30 prism diopters at near.

The classification of his strabismus is:

true divergence excess exotropia

simulated divergence excess exotropia


basic exotropia

convergence insufficiency exotropia

divergence excess exotropia

Your answer was CORRECT


Explanation
Divergence excess exotropia may be divided into two subtypes: true and pseudo. In the latter, the
turn appears larger at a distance because of enhanced tonic fusional convergence at near related to
accommodation. Interrupting fusion with prolonged occlusion (over 30 minutes) will cause the
deviation at near to increase to a measurement similar to the original distance measurement. This
is simulated divergence excess. If these manipulations have no effect (i.e. the distance
measurement is still larger), true divergence excess is said to exist.

If true divergence excess exists, the next important step clinically is determining the AC/A ratio.
This can be performed by re-measuring at near with a + 3.00 D. If the near deviation increases
close to the distance deviation with a + 3.00 D add, there is a high AC/A ratio contributing to the
distance:near discrepancy. This factor is important because these patients are prone to
overcorrection (75% overcorrection) if surgery for the full distance deviation is performed.
Patients (and parents) should be aware of the poor prognosis before surgery and of the possible
need for bifocals to decrease the high AC/A ratio postoperatively.

In young, healthy eyes, anterior segment ischemia becomes a concern after surgery on how many
rectus muscles?
2

Your answer was CORRECT


Explanation
Two muscles may nearly always be safely removed. Removal of four muscles is certain to cause
ischemia. Removal of three muscles is likely to lead to some degree of ischemia.

All of the following are typically associated with microtropia EXCEPT:


small angle manifest deviation
central suppression scotoma
abnormal retinal correspondence
myopia
reduced stereopsis

Your answer was CORRECT


Explanation
Most patients with microtropia are hypermetropic.

Microtropia is characterised by:


 small angle manifest deviation
 central suppression scotoma
 abnormal retinal correspondence
 anisometropia common
A 5-year-old boy presents with an exodeviation of 30 PD at distance and 10 PD at near on
alternate cover testing. A patch test is performed on this patient and he now measures 30 PD at
distance and 15 PD at near. What is the next most appropriate step?

dispense glasses prescription with bifocal

recess the lateral recti for an amount intermediate between the distance and near deviation

re-measure the deviation with a + 3.00 D add


recess the lateral recti for 30 PD.

Your answer was CORRECT


Explanation
This child exhibits true divergence excess. He needs to be measured with a + 3.00 D add to
determine if he has a high AC/A ratio. If the near deviation increases close to the distance
deviation with a + 3.00 D add, he has a high AC/A. This factor is important because these patients
are prone to overcorrection (75% overcorrection) if surgery for the full distance deviation is
performed. Parents should be aware of the poor prognosis before surgery and of the possible need
for bifocals to decrease the high AC/A ratio postoperatively.

All of the following are typical findings of infantile esotropia EXCEPT:


40 PD angle
+4.00D cycloplegic refraction
inferior oblique overaction
full doll's head eye movements
nystagmus on occluding one eye, with fast-phase towards the fixing eye

Your answer was CORRECT


Explanation
Characteristics of infantile esotropia:
 onset within first 6 months
 idiopathic
 large stable angle over 30PD
 alternating fixation
 cross fixation
 manifest-latent nystagmus
 normal refraction for age (+1.00 to +2.00D usually)
 inferior oblique overaction
 DVD (80% by 3 years)
 amblyopia (50%)
 Rx: surgery by 1 year, latest 2 years (only after amblyopia and refraction)
Parents bring their 3-year-old boy for examination having noticed a squint. It has been present
throughout the day since he was 2 years old. A cursory inspection reveals an obvious esotropia,
which appears larger when the child plays with an object in his hands. Cycloplegic refraction
reveals + 1.50D in both eyes.

Which of the following is FALSE?

the deviation at near is likely to be moderate, between 20 and 30 PD

the deviation at near is likely to be lessened with +3.00D lenses over each eye

an AC/A ratio of 4 is compatible with this case

the deviation at distance is not likely to be large

Your answer was CORRECT


Explanation
This is probably a case of non-refractive accommodative esotropia, with intermittent turn, worse
at near, normal refractive error and high AC/A ratio. Lenses of +3.00D for near work will relieve
the accommodative demand and prevent accommodative convergence from causing an esotropia.

A high AC/A ratio is over 5.

An exodeviation that is greater at distance than at near is known as:


true divergence excess exotropia
basic exotropia
convergence insufficiency exotropia
divergence excess exotropia
simulated divergence excess exotropia

Your answer was CORRECT


Explanation
Types of exotropia:
 Basic exotropia: exotropia equal at distance and near
 Convergence-insufficiency exotropia: exotropia greater at near
 Divergence-excess exotropia: exotropia greater at distance than near

Note: To further classify divergence excess exotropia as true or simulated further tests are needed,
in particular a 30 minute occlusion and re-testing for near and distance. This helps to get rid of any
tonic fusional convergence for near and elicits the full exotropia at near. If this test returns with
near and distance exotropia equal, then pseudo-divergence excess is present. If true divergence
excess is present, measuring AC/A ratio is another important test to perform, as the distance:near
discrepancy may be contributed to by a high AC/A ratio, which is important to know for surgical
planning and prognosis. Specifically, patients with true divergence excess and a high AC/A ratio
are prone to overcorrection (75% overcorrection) if surgery for the full distance deviation is
performed. Patients (and parents) should be aware of the poor prognosis before surgery and of the
possible need for bifocals to decrease the high AC/A ratio postoperatively.
Which of the following statements about cyclic esotropia is FALSE?

amblyopia is possible but relatively uncommon

the angle of deviation is generally moderate

full hyperopic correction may correct the esotropia

the age of onset is similar to accommodative esotropia

the cycle consists of 1 week of orthotropia and 1 week of esotropia

Your answer was CORRECT


Explanation
The cycle is typically 48 hours long: 1 day of esotropia, and 1 day of orthotropia. Many experts
believe that this is a variety of accommodative esotropia, with many features similar to it.

When tested with a Maddox rod held over the affected eye with its cylinders running horizontally,
a patient with new-onset excyclotropia will perceive:

an oblique line running superotemporal to inferonasal

a vertical line

a curved line concave toward the nose

a horizontal line

an oblique line running superonasal to inferotemporal

Your answer was CORRECT


Explanation
In excyclotropia, the superonasal retinal quadrant is rotated vertically toward the 12-o'clock
position, and the inferotemporal quadrant is rotated toward the 6-o'clock position. The vertical line
from the Maddox rod will run superonasal to inferotemporal on the retina and be perceived the
same way.

To distinguish true divergence excess exotropia from pseudo-divergence excess exotropia:

the AC/A ratio is calculated by the heterophoria method

the deviations are re-measured after prolonged monocular occlusion

the deviations are re-measured after cycloplegia

the AC/A ratio is calculated by the gradient method

Your answer was CORRECT


Explanation
Divergence excess exotropia may be divided into two subtypes: true and pseudo. In the latter, the
outward deviation is larger at distance because of enhanced tonic fusional convergence at near
related to accommodation. Interrupting fusion with prolonged occlusion (over 30 minutes) will
cause the deviation at near to increase to a measurement similar to the original distance
measurement. This is simulated divergence excess. If these manipulations have no effect (i.e. the
distance measurement is still larger), true divergence excess is said to exist.

If true divergence excess exists, the next important step is determining the AC/A ratio. This can be
measured by comparing the angle at near with and without a + 3.00 D lens. If the near deviation
increases close to the distance deviation with a + 3.00 D add, there is a high AC/A ratio
contributing to the distance:near discrepancy. This factor is important because these patients are
prone to overcorrection (75% overcorrection) if surgery for the full distance deviation is
performed. Patients (and parents) should be aware of the poor prognosis before surgery and of the
possible need for bifocals to decrease the high AC/A ratio postoperatively.

A patient has severe, grade 4 inferior oblique overaction. Which of the following procedures is
MOST likely to be helpful:
inferior oblique myomectomy
inferior oblique recession of 8mm
anteriorisation of the tendon to Marshall Parks point
inferior oblique disinsertion

Your answer was CORRECT


Explanation
Correction of inferior oblique overaction:
 Mild
o IO recession 8-10mm
 Moderate
o IO myomectomy
o IO disinsertion
 Severe
o extirpation of IO muscle
o denervation
o anteriorisation of the tendon to Marshall Parks point 3mm lateral to lateral border of
inferior rectus insertion and 1mm behind

An 8-year-old girl is found on routine optician assessment to have restriction of eye movements.
Specifically, there is limited elevation in adduction of the right eye, while elevation in abduction is
full. There is over-action of the left superior rectus but otherwise eye movements are normal.
Bielschowsky head tilt is negative.

What is the most likely diagnosis?

Right Duane’s syndrome

Right superior rectus restriction


Right inferior oblique palsy

Right Brown’s syndrome

Your answer was CORRECT


Explanation
Brown’s syndrome is characterised by limited elevation in adduction due to a restriction of the
superior oblique tendon. There is over-action of the contralateral superior rectus, but no other
muscle sequelae. The most important differential in a Brown’s syndrome is inferior oblique palsy.
However, in the latter there is superior oblique over-action, full muscle sequelae, and head tilt is
positive.

This question came in the 2014 FRCOphth.

On alternate-cover testing of a patient, with the left eye covered, the right eye fixes on a distance
target. As the cover is shifted to the right eye, the left eye moves down to pick up fixation. As the
cover is shifted back over the left eye, the right does not move in order to reassume fixation.

This set of findings is most consistent with:

right hyperdeviation

left hyperdeviation

overaction of the inferior obliques

overaction of the superior obliques

dissociated vertical deviation

Your answer was CORRECT


Explanation
Dissociated vertical deviation simulates a hyperphoria but violates Hering's law. When covered,
the left eye drifts up. When uncovered, the left eye moves down to assume fixation. By Hering's
law, the yoke muscles of the right eye, depressors, should receive equal innervation as the left eye
moves down. The right eye should be deviated downward under cover and move up to reassume
fixation when the cover is shifted back to the left eye. The dissociation of DVD refers to the
violation of Hering's law with dissociation of yoke muscles.

Which statement regarding strabismus surgery is TRUE:

the inferior rectus is the shortest extraocular muscle

the lateral rectus has no intermuscular septum connection to the oblique muscles

lower eyelid retraction can occur after recession of more than 4mm of inferior rectus
recession of superior rectus of 10mm is likely to significantly change upper lid position

Your answer was CORRECT


Explanation
The medial rectus is the shortest extraocular muscle. It is also the only extraocular muscle without
an intermuscular septum connection to an oblique muscle - hence its tendency if it slips during
surgery to retract far posteriorly within its capsule and be difficult to locate.

The inferior rectus has a strong connection to the lower lid, which can result in lower eyelid
retraction with recession of 4mm or more, especially in patients with thyroid eye disease and pre-
existing lid retraction.

Superior rectus recession can retract the upper lid, but it is less likely than the corresponding
phenomenon in the lower lid. Changes are more likely to be subtle than to significantly alter lid
position.

All of the following findings on examination of a 9-month-old infant are in keeping with
congenital or essential esotropia EXCEPT:
cycloplegic refraction of +2.00DS in both eyes
asymmetrical optokinetic responses
bilateral inferior oblique overaction
esotropia of 40PD on prism reflection testing for both distance and near
nystagmus with a null-point in chin down positioning

Your answer was CORRECT


Explanation
Congenital esotropia is associated with manifest-latent nystagmus. The null point in manifest-
latent nystagmus is always in adduction, with a resulting face turn towards the side of the fixing
eye. This is in contrast to congenital nystagmus (not associated specifically with congenital
esotropia), where the null point can be in any position of gaze, leading to individualised head
postures for patients depending on where their null point occurs.

Characteristics of manifest-latent nystagmus:


 bilateral
 becomes manifest when one eye occluded, blurred or intermittently suppressed
 jerk-type nystagmus
 null point in adduction
 fast phase towards fixing eye
 face turn towards the fixing eye as this dampens nystagmus
 associated with interruptions to binocular development: congenital esotropia, but also
monocular congenital cataracts
Characteristics of congenital nystagmus:
 jerk or pendular nystagmus
 no change in nystagmus with unilateral occlusion or blurring
 fast phase switches: to right in right-gaze, to left in left-gaze
 null point can occur in any position of gaze
 compensatory head postures vary by patient: face-turns either way or chin up, chin down
depending on the position of the null point
 titubation
 tends to dampen with convergence, darkness, sleep, when eye is covered
 increases with fixation
 paradoxical OKN response
Characteristics of infantile esotropia:
 onset within first 6 months
 idiopathic
 large stable angle over 30PD
 alternating fixation
 cross fixation
 manifest-latent nystagmus
 normal refraction for age (+1.00 to +2.00D usually)
 inferior oblique overaction
 DVD (80% by 3 years)
 amblyopia (50%)
 asymmetric monocular OKN response (favour objects moving temporally to nasally)
 Rx: surgery by 1 year, latest 2 years (only after amblyopia and refraction)

A 3-year-old boy has intermittent squint noted by his parents. Alternate cover test shows distance
exotropia with good recovery. Refraction: +3.00DS in both eyes. Prism cover test with +3.00DS
glasses shows 10 PD BI for near and 45 PD BI for distance.

What is the most likely diagnosis?


secondary exotropia

near exotropia

simulated intermittent distance exotropia

consecutive exotropia

true intermittent distance exotropia

Your answer was INCORRECT


Explanation
This case represents true intermittent distance exotropia because the angle for near remains
significantly less than the distance angle even when glasses are worn to prevent accommodative
convergence. In simulated distance exotropia, a high AC:A ratio means the angle for near is less
than the angle for distance.

Which of the following is FALSE:


peripheral suppression relieves diplopia more than confusion

stereopsis is typically normal in patients with abnormal retinal correspondence

suppression which alternates is less likely to cause amblyopia

abnormal retinal correspondence allows binocular single vision in the presence of heterophoria

Your answer was CORRECT


Explanation
Abnormal retinal correspondence allows some crude degree of binocular single vision in the
presence of heterophoria but stereopsis is usually absent.

Generally speaking, central suppression avoids confusion, while peripheral suppression avoids
diplopia. Suppression which is monocular leads to amblyopia, while suppression that alternates
eyes may not be associated with amblyopia.

The primary intorter of the globe in primary position is the:

superior rectus

levator muscle.

inferior oblique

superior oblique

inferior rectus

Your answer was CORRECT


Explanation
The superior muscles act as intorters, whereas the inferior muscles act as extorters. The oblique
muscles are more important for torsion than the vertical recti. Therefore, the primary intorter of
the globe in primary gaze is the superior oblique.

The site of action of botulinum toxin type A (Botox) is the:


motor nerve terminal, inhibiting acetylcholine release
motor nerve terminal, promoting cholinesterase release
sarcolemma of striated muscle, inhibiting acetylcholine release
sarcolemma of striated muscle, promoting cholinesterase release

Your answer was CORRECT


Explanation
Botulinum toxin type A inhibits acetylcholine release by binding to receptors of the motor nerve
terminals. The denervation that results usually lasts several months. Some patients develop
antibodies to the medication, resulting in a decreased efficacy of the botulinum toxin with
repeated administration.

Applications of botox in ophthalmology include:


 benign essential blepharospasm
 hemifacial spasm
 paralytic strabismus
 torticollis

All of the following tests of ocular alignment require fixation in the deviated eye for quantification
of the angle of strabismus EXCEPT:
the cover-uncover test with prisms

the simultaneous prism-cover test

the Krimsky test


the alternate-cover test with prisms

Your answer was CORRECT


Explanation

The Krimsky test (shown above) uses prisms over the fixating eye to center the light reflex over
the pupil in the deviating eye. Foveal fixation is not required in the deviated eye.

In a gentleman with Brown's syndrome, which of the following in the past medical history is
LEAST likely to be implicated as having a role in causing his condition?
congenital sixth nerve palsy
previous scleral buckle for retinal detachment
Marfan syndrome
acromegaly
rheumatoid arthritis

Your answer was CORRECT


Explanation
Brown's syndrome may be:
 congenital
o short tendon
o tight trochlear
o nodule on the superior oblique tendon
 acquired
o trauma
o tenosynovitis (rheumatoid arthritis)
o Marfan's
o acromegaly
o extraocular surgery (e.g. buckle)

A 9-year-old boy has left esotropia of 7 prism dioptres. His acuities are 6/6 OD and 6/9 OS.
Visioscopy in the left eye suggests eccentric fixation.

Which of the following responses is LEAST likely to occur with the Worth four-dot test? (green
filter OD and red filter OS)

he sees all four lights for near fixation

he sees two red lights for distance fixation with +5D lens OD

he sees three green lights for distance fixation

he sees two red lights for near fixation

Your answer was CORRECT


Explanation
The patient has left monofixation syndrome (microtropia), with the right eye used to fixate
distance objects while the same image falls within the suppression area of the left fovea. As the
Worth 4-dot flashlight is brought closer to the patient, the dots begin to project onto peripheral
retina outside the central monofixation scotoma until a fusion response (4 lights) is obtained. This
usually occurs between 2 and 3 ft.

Which statement is FALSE regarding the synoptophore:

if the subjective angle is less than the objective angle but not zero there is unharmonious ARC
it can measure horizontal, vertical and torsional misalignments

if the objective angle is equal to the subjective angle there is harmonious ARC
it can test for stereopsis

Your answer was CORRECT


Explanation
The synoptophore can measure all aspects of binocular single vision including simultaneous
perception, fusion (including range of fusion) and stereopsis. It can also measure the degree of
misalignment for horizontal, vertical and torsional misalignments in all directions of gaze. It can
detect suppression and abnormal retinal correspondence (ARC).

To detect ARC with the synoptophore, the objective angle (OA) and subjective angle (SA) are
measured, which gives the angle of anomaly (AOA).

AOA = OA - SA

In normal retinal correspondence (NRC), the SA is equal to OA and the AOA will be zero.

In unharmonious ARC, the SA will be less than the OA (but the SA will not be zero)

In harmonious ARC, the SA will be zero, so the AOA will be equal to the OA.

All of the following examination findings are characteristic of Brown's syndrome EXCEPT:

ipsilateral superior oblique overaction


slight hypotropia in primary position

normal elevation in abduction

V pattern

downshoot in adduction

Your answer was CORRECT


Explanation
In Brown's syndrome, there is contralateral superior rectus overaction, but there is no ipsilateral
superior oblique overaction. This helps to differentiate Brown's syndrome from inferior oblique
palsy, where there is superior oblique overaction.

Findings Brown's syndrome IO palsy


Hypotropia in primary position slight significant

Muscle Sequelae no ipsilateral SO overaction ipsilateral SO overaction

Alphabet Pattern V pattern A pattern

Head Posture slight marked

Forced Duction Test positive negative

The findings below are all consistent with a patient who has a right microtropia with identity
EXCEPT:
reduced stereopsis
flick movement on cover test
right eccentric fixation
no movement when a 4 PD base-out prism is placed before the right eye
anisometropia

Your answer was CORRECT


Explanation
Microtropia with identity describes those patients with no manifest movement on cover test, as the
eccentric fixation point coincides precisely with the angle of abnormal retinal correspondence.

Microtropia is characterised by:


 small angle manifest deviation less than 10PD
 central suppression scotoma
 eccentric fixation
 abnormal retinal correspondence
 anisometropia common - often hypermetropia +- astigmatism
 stereopsis reduced
 amblopia
 Rx: refractive correction, occlusion for amblyopia

The maximal advisable recession of the medial rectus muscle in the initial surgical management
for esotropia is:
3 mm
5 mm
4 mm
6 mm
5.5 mm

Your answer was CORRECT


Explanation
If this results in undercorrection, then additional surgery is necessary.

Maximal advisable initial surgery:


 Esotropia:
o medial recession: 6 mm
o lateral resection: 9 mm
 Exotropia:
o medial resection: 10 mm
o lateral recession: 10 mm

Which is TRUE of nystagmus blockage syndrome?

the esotropia is due to an excess of accommodation


the squinting eye remains adducted when a base-out prism is introduced in front of the
fixing eye

the nystagmus is absent when the eyes are closed

pupil mydriasis may occur during the esotropic phase

Your answer was CORRECT


Explanation
The esotropia in nystagmus blockage is due to the convergence mechanism attempting to suppress
congenital nystagmus, hence the prism has little effect on the squinting eye as it simply drives
further convergence.

The nystagmus is present and remains the same whether both eyes are open or one is covered. It
may be dampened but is not eliminated completely by closing both eyes, by sleep and by
convergence.

The esotropia is non-accomodative and variable.

Amblyopia is common and the esotropia usually unilateral.

The patient develops a face-turn to the side of the fixing eye.

Pupil miosis may occur during the esotropic phase.

Nystagmus blockage syndrome has a high association with neurological disorders and ocular or
oculo-cuanteous albinism. The outcome of surgery is unfortunately unpredictable.

The entity in the differential diagnosis with spasmus nutans that must be ruled out is:
parasellar glioma
syringomyelia
optic nerve meningioma
pontine glioma
cerebellar astrocytoma

Your answer was CORRECT


Explanation
A similar clinical entity to spasmus nutans can occur with parasellar gliomas, where there is a
monocular (or asymmetric) nystagmus. It is important to consider this differential in any child
with spasmus nutans.

Spasmus nutans is characterised by:


 onset 6 months to 3 years of age
 idiopathic and self-limiting (usually within 2 years)
 rapid, small-amplitude disconjugate jerk nystagmus
 head nodding
 torticollis
 nystagmus may appear entirely monocular and raise the specter of chiasmal or hypothalamic
glioma, which needs always to be excluded

Which of the following is a feature of fully accommodative esotropia?

the difference in the deviation between distance and near is usually over 15PD

amblyopia is usually present

the esotropia is fully corrected by a myopic correction

the AC/A ratio is normal

Your answer was CORRECT


Explanation
Patients with fully accommodative esotropia have an esotropia which is fully corrected with a
hypermetropic correction. They are relatively hypermetropic for their age, the AC/A ratio is
normal and they usually have good binocular vision. The prognosis is generally better than for
partially accommodative esotropia.

Which is FALSE regarding stereopsis:

it is non-existent on the horopter

it is not present at the point of focus

it is minimal beyond 15 degrees

vertical and torsional image disparity play an important role in stereopsis

Your answer was CORRECT


Explanation
Stereopsis is caused by horizontal retinal image disparity. Vertical and torsional disparity do not
contribute to stereopsis. Stereopsis occurs only within Panum's space. Stereoacuity increases as
the horopter is approached but is zero on the horopter plane (which includes the point of focus)
because corresponding retinal points are stimulated.

A 1-year-old child is diagnosed with anisometropic amblyopia. Cycloplegic refraction +6.00DS


right eye, +3.00DS left eye. The child has a 25PD right esotropia for near and distance.

Which of the following treatment strategies in this case is NOT appropriate?

if no progress is made after 3 consecutive months of patching, it is appropriate to institute


pharmacological penalisation
if compliance with patching is poor, an alternative is to fully correct the right eye refractive error
while undercorrecting the left eye

occlusion therapy should be instituted prior to surgery

spectacle correction and full-time occlusion of the left eye for 2 weeks with close orthoptic
review in 2 weeks

Your answer was CORRECT


Explanation
Full time occlusion should not exceed 1 week per year of age. Other statements above are true.

A 3-year-old child presents with intermittent exotropia. All of the following would most likely
describe her condition EXCEPT:
large convergence amplitudes
suppression
amblyopia
excellent stereopsis

Your answer was CORRECT


Explanation
In intermittent exotropia there is:
 suppression when eyes are deviated
 excellent stereoacuity when eyes aligned
 amblyopia is rare
 large convergence amplitudes

The neural pathway for a saccade to the right originates in the:


left parieto-occipital lobe

right frontal lobe

right parieto-occipital lobe

left frontal lobe

Your answer was CORRECT


Explanation
Saccades are controlled by the contralateral frontal lobe.
Pursuit is controlled by the ipsilateral parietal lobe.
A patient presents urgently to the eye department via accident and emergency. You have not met
the patient and there is no history available as yet, since the patient is being seen first by the
orthoptist. The orthoptist report reads: there is diplopia in primary position. On laevoversion, there
is severe restriction of adduction of the right eye which does not improve on covering his left eye.
Right vertical movements are mildly reduced. The left eye has full extraocular motility

These findings fit most precisely with which diagnosis?


consecutive exotropia
bilateral internuclear ophthalmoplegia
lateral rectus entrapment
Duane's syndrome Huber type 2

Your answer was CORRECT


Explanation
In mechanical restrictions the ductions are the same as the versions.

Mechanical restriction can be due to:


 trauma with muscle entrapment
 orbital mass
 tethering of optic nerve
 Duanes syndrome
 restrictive myopathies
 thyroid eye disease
Although Duane's can cause a mechanical restriction, the history of recent-onset via A&E,
diplopia in primary position and the clinical findings are most in keeping with entrapment from
trauma.

Which one of the following provides an EXCEPTION to Hering's law?

Duane's type I

alternating esotropia

dissociated vertical deviation


convergence

Your answer was CORRECT


Explanation
Hering's law states that when an extraocular muscle receives stimulation, its yoke muscle (the
prime mover in the contralateral eye in the same field of gaze) receives equal innervation. An
exception to Hering's law is DVD. In DVD, one eye elevates, extorts, and abducts without any
innervation to the contralateral eye.

Sherrington's law states that when one extraocular muscle is stimulated, the ipsilateral antagonist
is inhibited. In Duane's type I, the lateral rectus muscle is innervated by part of the medial rectus
subdivision of the third nerve. This may be a result of a congenital agenesis of the abducens
nucleus, which has been demonstrated pathologically. As a result of this aberrant innervation
when the medial rectus is stimulated to contract, the lateral rectus also receives stimulatory
impulses, thus violating Sherrington's law.

All of the following findings are characteristic for a right Type 2 Duane's syndrome EXCEPT:

binocular single vision in primary position

upshoot of the right eye on adduction

right palpebral fissure narrowing on adduction more than abduction

esotropia in primary position

deficiency of convergence

Your answer was CORRECT


Explanation
Type 2 Duane's is associated with exotropia (or straight) in primary position, while type 1 and
type 3 are associated with esotropia (or straight) in primary position.

All of the following are late clinical findings consistent with an inferior blowout fracture of the
orbit EXCEPT:

ipsilateral hypotropia on upgaze

paraesthesia or hypaesthesia of the infraorbital region

ipsilateral hypertropia on downgaze

positive forced ductions

proptosis

Your answer was CORRECT


Explanation
Figure: orbital floor blowout fracture with inferior rectus entrapment.

Enophthalmos is frequently cosmetically unacceptable. In the acute setting, there may be


proptosis, but this usually gives way to enophthalmos as swelling subsides. Restriction is often
apparent in both upgaze and downgaze.
An 8-year-old boy presents with the ocular motility shown above. Which one of the following is
NOT true of this condition?

this condition often has inferior oblique overaction

early onset of this condition can result in facial asymmetry

the patient has a compensatory head tilt away from the pathology

patients with this condition have normal vertical fusion amplitudes

Your answer was CORRECT


Explanation
This patient has congenital fourth nerve palsy. Longstanding fourth nerve palsy results in ocular
torticollis with a compensatory head tilt away from the side of the palsy that can lead to facial
asymmetry of the dependent side.

These patients have large vertical fusional amplitudes that help differentiate them from acquired
fourth nerve palsies. (Normal vertical fusional amplitude is 2-3 prism dioptres)

Regarding childhood strabismus, which statement is FALSE:


a monofixation syndrome with peripheral fusion, good cosmesis, and limited stereopsis is a fairly
common outcome of successful management of infantile esotropia

refractive, accommodative patients where esotropia is fully corrected by hyperopic glasses


are more likely to develop stress-induced decompensation in their esotropia than patients
with a high AC/A ratio
sensory deprivation in a child younger than 6 years is more likely to lead to esotropia, whereas
that occurring in adults is more likely to lead to exotropia

consecutive exotropia following surgery for esotropia should be initially observed, with patching
if necessary, as a majority resolve within weeks to month after surgery

Your answer was CORRECT


Explanation
With illness or stress, non-refractive accommodative esotropia (high AC/A ratio) are more likely
to decompensate than the refractive variety.

Which one of the following statements about overaction of the superior oblique muscle is FALSE?

it may be associated with an exotropia in primary gaze

unilateral cases may develop an ipsilateral hypotropia in primary gaze

many cases are secondary to weakness of the ipsilateral inferior oblique muscle
it is frequently associated with exotropia in downgaze

the hallmark is depression on attempted adduction

Your answer was CORRECT


Explanation
Overaction of the inferior oblique may be seen as a primary disorder or secondary to underaction
of the ipsilateral, antagonist superior oblique. This duality of causes for superior oblique
overaction is not seen. Because isolated underaction of the inferior obliques is quite uncommon,
secondary overaction of the superior oblique is also uncommon. Only primary mechanisms are
recognised.

Having agreed to undertake surgery, which of the following factors is MOST crucial in choosing
the appropriate technique for correcting an A- or V-pattern deviation:

the chronicity of the deviation

presence or absence of head tilt

presence and type of oblique muscle dysfunction


presence or absence of peripheral fusion
Your answer was CORRECT
Explanation
If there is no significant oblique dysfunction, then transposition of the rectus insertions should be
performed to correct the A or V patterns. If there is significant oblique dysfunction, then surgery
on the obliques is preferred.

A large angle kappa is consistent with all of the following EXCEPT:


familial exudative vitreoretinopathy
persistent posterior fetal vasculature
very high myopia
combined hamartoma of the retina and RPE
retinopathy of prematurity

Your answer was CORRECT


Explanation
High myopia is likely to cause a negative angle kappa, which leads to pseudo-esotropia. A large
angle kappa causes a pseudo-exotropia.

A large angle kappa occurs when there is temporal displacement of the macula, as can occur with:
 ROP
 FEVR
 combined hamartoma of RPE and retina
 persistent posterior fetal vasculature

The purpose of the prism adaptation test is to determine whether a patient has:
potential for stereoposis

the ability to achieve sensory fusion

a restrictive or paretic defect

a stable deviation over time

Your answer was CORRECT


Explanation
The prism adaptation test involves prescription of corrective prisms prior to undertaking muscle
surgery in order to determine whether the patient can achieve sensory fusion with alignment.
Note: This test may be associated with an increase in the angle of strabismus because the patient
becomes accustomed to the prisms.
A 4-year-old has sensorineural deafness, a short neck, low hairline at the back of the head, and a
high scapula.

What oculomotility finding is MOST likely to be associated with these findings?

restriction of upgaze in adduction

lid lag

Cogan's twich

ptosis with restriction of adduction, elevation and depression

narrowing of palpebral fissure on adduction

Your answer was CORRECT


Explanation
This patient has Klippel-Feil syndrome, a rare defect of fusion of the cervical spine, associated
with vertebral and skeletal deformities. Klippel-Feil syndrome may be associated with deafness
and Duane's syndrome, a triad called Wildervanck's syndrome.

A leftward pursuit movement originates in the:


left frontal lobe

right frontal lobe

right parieto-occipital lobe

left parieto-occipital lobe

Your answer was CORRECT


Explanation
Pursuit is controlled by the ipsilateral parietal lobe (sometimes considered parieto-occipital lobe as
the area is on the border).

Saccades are controlled by the contralateral frontal lobe.

The muscle most commonly lost in strabismus surgery is the:

inferior oblique

superior oblique

medial rectus

lateral rectus
Your answer was CORRECT
Explanation
The medial rectus is the most commonly lost in strabismus surgery, because it is under the greatest
tension, and tends to retract to the orbital apex where it is difficult to recover.

Surgical strategies for the management of a right superior oblique paresis with symptomatic
diplopia include all of the following EXCEPT:

right superior oblique tuck

right superior rectus recession

right inferior rectus recession


left superior oblique tenectomy

right inferior oblique myectomy

Your answer was CORRECT


Explanation
Surgical management of superior oblique underaction is generally aimed at weakening the
ipsilateral inferior oblique (myectomy or disinsertion) and/or strengthening the ipsilateral superior
oblique (tuck). If the hyperdeviation is greater than 35 prism diopters in primary gaze, then
strengthening the ipsilateral inferior rectus or weakening the ipsilateral superior rectus may be of
value. Contralateral superior oblique weakening also may be useful in certain cases.

All of the following statements about dissociated vertical deviation are true EXCEPT:
it is often bilateral
it is frequently associated with congenital esotropia
the deviation is highly consistent in measurements
it can be made manifest by monocular vision loss

Your answer was CORRECT


Explanation
The deviation in DVD is highly variable and difficult to measure, making surgery difficult to
quantify accurately.

Dissociated vertical deviation:


 present in 60% to 80% of patients with congenital esotropia
 usually bilateral and asymmetric
 aetiology unknown
 probably due to early disruption of binocular development
 high-grade stereopsis and bifoveal fixation are not seen
 during visual inattention, the non-fixating eye slowly drifts up, extorts, and abducts without a
corresponding hypotropia of the fellow eye on alternate cover testing
 thus, it does not obey Hering's law

A 12-year-old girl has symptomatic convergence-excess esotropia. She is not keen on surgery or
Botox.

Which of the following is the most appropriate conservative treatment:

base-out prisms

atropine drops

pilocarpine drops

patching

bifocals

Your answer was CORRECT


Explanation
The best conservative treatment for convergence-excess esotropia is bifocal glasses to reduce the
accommodative drive to convergence.

A 25-year-old develops right abducens palsy following a road traffic accident with head trauma.
He has a right esotropia and is tested with the Lancaster red-green test. He wears the goggles with
the green lens over the right eye and the red lens over the left eye. The examiner holds the green
light as a fixation target centrally, and the patient moves the red light.

The examiner will observe:


the red light to the right of the green light and the magnitude of separation of lights will be the
same if the googles were reversed

the red light to the left of the green light and the magnitude of separation will be greater than if the
test was performed with the goggles reversed

the red light to the left of the green light and the magnitude of separation of lights will be the same
if the googles were reversed

the red light to the right of the green light and the magnitude of separation will be greater
than if the test was performed with the goggles reversed

Your answer was CORRECT


Explanation
In the Lancaster red-green test, the fovea of each eye is isolated with duochrome glasses. The eye
under green glass (right) will not see the red light and the eye under the red glass (left) will not see
the green light. By holding the green light centrally, the examiner holds the patient's right fovea
centrally. Assuming normal retinal correspondence (NRC), the patient will direct the red light in
space along his left visual axis to place the red light on his left fovea and superimpose the foveal
images. The patient's left visual axis (in esotropia) crosses his right visual axis, so the left foveal
image will fall to the right of the right foveal image, as seen by the examiner. To the patient, the
images appear superimposed (assuming NRC).
In this case, the deviated right eye is fixing centrally (green glass). The deviation produced
(secondary deviation) is larger than if the non-paralytic eye is fixing.

All are true of the Maddox rod EXCEPT:

it is useful in the detection of macular disease

it is used to determine the degree of latent deviation for distance

it is composed of cylindrical lenses

it produces a straight line in the plane of the long axis of the lenses

Your answer was CORRECT


Explanation
A Maddox rod forms an image perpendicular to the plane of the long axis of the lenses. All other
statements are true.

A 27-year-old man with moderate hyperopia presents for routine examination. There is a 10-
prism-diopter alternating esotropia at distance. While reading through his distance correction at 20
cm, there is a 35-prism-diopter esotropia. Eye movements are full, and he denies any history of
prior surgery.

Which of the following statements is TRUE:


he has the sequelae of pure, classic infantile esotropia
he probably will note double vision if questioned appropriately
he has a high accommodative convergence to accommodation ratio (AC/A)
he must have amblyopia in one eye
he probably has restrictive strabismus

Your answer was CORRECT


Explanation
A patient with an esotropia 10 prism dioptres (or more) greater at near than distance is said to have
a clinically high AC/A ratio.

AC/A ratio can be formerly calculated by:


 Heterophoria method:
[(near deviation – distance deviation)/accommodative demand]+interpupillary distance in cm
 Gradient method:
difference in deviation induced by a lens/accommodative gradient
 Clinical method:
if an accommodative esotropia is 10PD greater at near than distance AC/A ratio is said to be
clinically high
Note: A normal AC/A ratio is between 3 to 5.

A 46-year-old man presents to A&E with new-onset right esotropia. A red glass is placed over the
left eye and the patient is asked to fixate at a distant point-light target. The patient reports that he
only sees a red light.

Which statement is TRUE:


use of a vertical prism can determine if the patient has normal retinal correspondence
the patient must have anomalous retinal correspondence

the patient will have normal stereoacuity with his distance correction in place

the motility defect is recent in origin

Your answer was CORRECT


Explanation
The right image is probably being suppressed, in which case this is a longstanding esotropia,
perhaps only recently noticed or becoming more evident. There may be either normal retinal
correspondence (NRC) or anomalous retinal correspondence (ARC). To determine which, the
right image can be moved out of the central suppression scotoma with a vertical prism. In ARC,
the right white light will appear directly below the left red light.

On the other hand, it is possible in this case that the patient is malingering (voluntary convergence
spasm) and being less than truthful about his perceptions. Findings that would be supportive of
this conclusion include a variable and unpredictable amount of esotropia in various gaze positions,
as well as noticeable miosis with lateral gaze, particularly to the right.

An 8-year-old has the following orthoptic report:

Visual acuity: 6/6 both eyes


Cover test: right esotropia
Prism cover test: BO 10PD for near; BO 40PD for distance
Ocular motility: incomitant deviation
Refraction: RE +0.50D; LE +0.75D

These findings are most consistent with which diagnosis?

distance esotropia

partially accommodative esotropia

non-refractive convergence excess

paretic esotropia
near esotropia

Your answer was CORRECT


Explanation
The most significant finding in the report is incomitance of the deviation. Together with greater
deviation for distance than for near, this is highly suggestive of a paretic sixth nerve palsy.

Children and young adults can present with isolated sixth nerve palsy after viral infection or
trauma. A head scan is required, however, to rule out neoplasm e.g. brain glioma.

An 8-year-old has the following orthoptic report:

Visual acuity: 6/6 both eyes


Cover test: right esotropia
Prism cover test: BO 10PD for near; BO 40PD for distance
Ocular motility: incomitant deviation
Refraction: RE +0.50D; LE +0.75D

These findings are most consistent with which diagnosis?

distance esotropia

partially accommodative esotropia

non-refractive convergence excess

paretic esotropia
near esotropia

Your answer was CORRECT


Explanation
The most significant finding in the report is incomitance of the deviation. Together with greater
deviation for distance than for near, this is highly suggestive of a paretic sixth nerve palsy.

Children and young adults can present with isolated sixth nerve palsy after viral infection or
trauma. A head scan is required, however, to rule out neoplasm e.g. brain glioma.

Which of the following statements about infantile or congenital esotropia is FALSE?


the esotropia is usually large, over 30 prism diopters
amblyopia is less dense if the child crossfixates
hypermetropia is usually between +3.00 and +6.00
it is generally seen in isolation but may be associated with neurologic abnormalities
Your answer was CORRECT
Explanation
Hypermetropia is usually low in infantile esotropia.

Congenital esotropia is characterised by:


 large angle esotropia, usually greater than 30 PD
 onset usually during the first few months, by definition by 6 months of age
 cross-fixation may be present
 latent nystagmus
 dissociated vertical deviation (in up to 60%-70%)
 inferior oblique overaction with V-pattern esotropia (in up to 60%-70%)
 mild hyperopia, + 1.00 to + 2.00 D

Which statement is TRUE regarding the treatment of esotropia?

accommodative esotropia is more likely to require surgical intervention than infantile esotropia

bifocals are most helpful in the management of patients with non-refractive accommodative
esotropia
accommodative esotropias rarely progress over the first 5 to 7 years of life

if refractive correction fails to solve the problem, the only solution is surgical

Your answer was CORRECT


Explanation
Bifocals are generally most helpful in non-refractive accommodative esotropia, where high AC/A
ratios make the esotropia worse at near. Other statements above are false.

Surgery is necessary in virtually all cases of infantile esotropia; however many cases of pure
accommodative esotropia will resolve with time and refractive correction. If full refractive
correction is not the solution, some experts advocate atropine penalization with full correction or
pilocarpine treatment to provide accommodation without convergence. These steps frequently fail,
however, and are controversial.

Accommodative esotropia can progress over the first 5 to 7 years and careful follow-up is
necessary.

um – Tutor Mode
Question 4 of 10
Score: 71 %
Which is TRUE about diplopia after strabismus surgery?

children are more likely to experience symptoms than adults


the complication, if persistent 2 weeks after surgery, suggests a second surgical procedure will be
required

a trial of preoperative prisms is helpful in predicting who is likely to suffer from this
complication
postoperative diplopia is likely to develop in undercorrection of intermittent exotropia

Your answer was CORRECT


Explanation
The prism adaptation test is indeed a valuable tool for predicting postoperative diplopia. In
congenital strabismus, suppression prevents diplopia so children are less likely to experience
diplopia compared to adults. Undercorrected intermittent exotropia will be associated with
suppression and a quick recurrence of large-angle turns. The most frustrating diplopia is
frequently caused by small-angle residual turns, which are manageable with prisms. Many cases
will resolve spontaneously over months.

A vertical slit is projected onto the fovea of the right eye, whereas a horizontal slit pattern is
projected onto the fovea of the left eye. The subject reports perceiving rapidly alternating images
of each pattern; first one and then the other, but never simultaneously.

This perception is an example of:

fusion

suppression

retinal rivalry

stereopsis

abnormal retinal correspondence

Your answer was CORRECT


Explanation
Rapid alternation of images suggests retinal rivalry. Preference of one image occurs in suppression

An 8-year-old child with Down's syndrome attends clinic with a squint. On Hirschberg testing,
you note that the corneal reflex in the right eye is displaced nasally and is falling just on the pupil
edge.

What is the approximate angle of deviation?


right exotropia of 15PD
right esotropia of 30 PD
right esotropia of 15 PD
right exotropia of 30 PD
Your answer was CORRECT
Explanation
Regarding the Hirschberg corneal reflection test:

1 mm displacement = approximately 7 degrees = 15 PD

Reflex displaced to:


 pupil edge = 2 mm = 15 degrees = 30 PD
 midway from pupil to limbus = 4mm = 30 degrees = 60PD
 limbus = 6mm = 45 degrees = 90PD

A 5-year-old boy presents with an exodeviation of 30 PD at distance and 10 PD at near on


alternate cover testing. A patch test is performed on this patient and he now measures 30 PD at
distance and 15 PD at near. The patient then undergoes a bilateral lateral rectus recession. One
week later, he measures 10 PD consecutive esotropia. The child is complaining of diplopia.

What is the next most appropriate step?

start penalization OD

start full-time patch OD

start alternate patching

observe and have the patient return in 6 weeks

Your answer was CORRECT


Explanation
A small esotropia of 8 to 15 PD postoperatively is desirable, with 20 PD being the upper limit of
acceptance. Post-operative diplopia associated with an initial overcorrection is normal and usually
resolves by 1 to 2 weeks. In younger children (less than 4 years old), part-time alternate patching
helps prevent the development of suppression.

For a patient with accommodative esotropia, a greater angle for near of how many prism dioptres
establishes a clinically high AC/A ratio?

high AC/A ratio cannot be determined by this method

30 prism dioptres

10 prism dioptres
20 prism dioptres

Your answer was CORRECT


Explanation
An esotropia 10 prism dioptres (or more) greater at near than distance establishes a clinically high
AC/A ratio.

Amblyopia will be LEAST likely to occur in which setting?


infant with +4.00 right eye and +2.00D left eye
infant with +5.00D both eyes
infant with +3.00D spherical equivalent and cylinder of 1.00D
infant with -6.00D both eyes

Your answer was CORRECT


Explanation
Refractive errors causing amblyopia:
 hyperopia over 4-5 D
 myopia over 6-7 D
 anisometropia: difference over 1.50 for hyperopes or 3.00D for myopes
 meridional: cyclinder over 1.50D
Note: amblyopia is less likely in myopia compared to an equivalent degree of hyperopia because
children with myopia can still focus clearly for near objects.

Which is LEAST likely to be considered in the differential diagnosis of early-onset (infantile)


esotropia?
Bilateral sixth nerve palsy
Mobius syndrome
Duane Type 2
Nystagmus blockage syndrome
Strabismus fixus

Your answer was CORRECT


Explanation
Duane Type 2 is associated with exotropia (not esotropia).

The differential of early-onset esotropia includes:


 congenital sixth nerve palsy
 sensory esotropia
 nystagmus blockage syndrome
 Duane syndrome Type 1 and 3
 Mobius syndrome
 Strabismus fixus
A 3-year-old child with mixed mechanism esotropia has been treated for 9 months with full
hyperopic correction and occlusion therapy, and the decision is made for surgery. Visual acuity
measures 6/30 in the right eye and 6/7.5 in the left eye. There is an esotropia of 30 prism diopters
in all directions of gaze.

Appropriate surgical intervention would consist of:

bilateral medial rectus recession of 7 mm in both eyes

bilateral lateral rectus resection of 7 mm in both eyes

recession of the left medial rectus 4.5 mm and resection of the left lateral rectus 7 mm

bilateral medial rectus recession of 4.5 mm in both eyes

recession of the right medial rectus 4.5 mm and resection of the right lateral rectus 7 mm

Your answer was CORRECT


Explanation
Since there is amblyopia, a right-eyed procedure should be chosen (to minimise operative risks on
the better-seeing left eye). The guidelines for a 30-prism-diopter esotropia are 4.5 mm medial
rectus recession and 7 mm lateral rectus resection.

A patient has esotropia measured as 10 PD for both distance and near. On Bagolini striated
glasses, the patient reports seeing 2 intersecting diagonal lines at 90 degrees to each other
intersecting in the middle.

What is an appropriate interpretation of the test:

he has harmonious abnormal retinal correspondence


he has right suppression

he has functional visual impairment

he has left suppression

he has normal BSV

Your answer was INCORRECT


Explanation
The patient has a manifest deviation yet Bagolini test is normal, with intersection of the two
diagonal lines in the middle. This suggests harmonious abnormal retinal correspondence (ARC).

Which statement is FALSE regarding infantile esotropia?


the angle of deviation is similar for near and distance
latent nystagmus is usually present
it is associated with dissociated vertical deviation
patients with cross fixation should have occlusion therapy

Your answer was INCORRECT


Explanation
Patients with cross-fixation usually have similar visual acuity in both eyes; thus occlusion therapy
is usually not indicated in this setting.

Congenital (infantile) esotropia is characterised by:


 large angle esotropia, usually greater than 30 PD
 onset usually during the first few months, by definition by 6 months of age
 cross-fixation may be present
 latent nystagmus
 dissociated vertical deviation (in up to 60%-70%)
 inferior oblique overaction with V-pattern esotropia (in up to 60%-70%)
 mild hyperopia, + 1.00 to + 2.00 D

A bilateral inferior oblique myectomy to correct a V-pattern exotropia, can be expected to yield
how much esodeviation in upgaze?

50 prism diopters

25 prism diopters

40 prism diopters

5 prism diopters

10 prism diopters

Your answer was CORRECT


Explanation
A bilateral inferior oblique myectomy produces 15 to 25 prism diopters of esodeviation in upgaze
(to correct a V pattern).

A bilateral superior oblique tenotomy causes 40 prism dioptres of esodeviation in downgaze (to
correct an A pattern).

A 6-year-old boy presents with an exodeviation of 30 PD at distance and 10 PD at near on


alternate cover testing. What is the next most appropriate step?

recess the lateral recti for an amount intermediate between the distance and near deviation

recess the lateral recti for 30 PD

occlude one eye for 30 minutes and remeasure the deviation


dispense glasses prescription with bifocal

Your answer was CORRECT


Explanation
This patient exhibits a difference of greater than 10 PD between near and distance. The patch test
needs to be performed to differentiate a pseudo-divergence excess from a true divergence excess.
This differentiation helps when deciding on the amount of surgery to perform and on the
prognosis. The patient is monocularly occluded for 30 to 60 minutes and re-measured without
letting him restore binocular fusion. The patch dissociates the eyes to suspend all tonic fusional
convergence and to reveal the full latent deviation at near, eliminating the distance/near disparity
if pseudo-divergence excess is present.

Bifocals are appropriate for a patient with which one of the following measurements?

Distance ortho wearing full distance correction of + 2.00 DS OU and Near ET15 wearing full
distance correction of + 2.00 DS OU

Distance ET10 wearing full cycloplegic correction of - 2.50 DS OU and near ET13 wearing full
cycloplegic correction of - 2.50 DS OU

Distance ET20 first time visit, cycloplegic retinoscopy + 4.00 DS OU

Distance ET30 wearing the full distance correction of + 4.00 DS OU and near ET45 wearing the
full distance correction of + 4.00 DS OU

Your answer was CORRECT


Explanation
The indication for prescribing bifocals to treat esotropia is specific. The patient must be able to
fuse in the distance and be esotropic at near while wearing the full hyperopic correction. Giving a
bifocal to a patient who has a deviation at distance despite full hyperopic correction will have no
effect on the distance deviation. This patient needs surgery.

A 6-year-old has severe limitation of horizontal gaze. Both eyes are orthophoric in the primary
position. You note that downgaze is normal and Bell's phenomenon is preserved.

What is the most likely diagnosis?

Moebius syndrome
double elevator palsy

strabismus fixus

congenital fibrosis of the extraocular muscles

Your answer was CORRECT


Explanation
Moebius syndrome can be caused by bilateral 6th nerve palsies or by horizontal gaze palsy. The
vertical gaze is usually preserved to a greater extent than the horizontal gaze. Unlike congenital
fibrosis of the extraocular muscles, the Bell's phenomenon is usually preserved in Moebius
syndrome. Strabismus fixus is not a candidate in this case. It is a rare ocular motor abnormality in
which one or both eyes are anchored in an extreme adduction or abduction position. It may be
congenital or acquired; the congenital form may be a variant of congenital fibrosis of the
extraocular muscles.

Which extra-ocular muscle is located furthest from the limbus?


lateral
inferior
superior
medial

Your answer was CORRECT


Explanation
Insertions of the recti muscles:
 Medial: 5.5 mm
 Inferior: 6.5 mm
 Lateral: 6.9mm
 Superior: 7.7 mm

A mother brings her 3-year-old child for evaluation of esotropia, which has been noticed over the
past 2 months. On general inspection, you note prominent epicanthic folds and a broad nasal
bridge with symmetric corneal reflections.

Which of the following is MOST likely to be true?

with time, the condition will spontaneously improve

the esotropia is likely to be A-shaped rather than V-shaped

correcting with a full hypermetropic prescription may improve the condition

there is likely a family history of Duane's syndrome

dissociated vertical deviation is a recognised association

Your answer was CORRECT


Explanation
This child has pseudo-strabismus due to a broad nasal bridge with epicanthic folds that obscure
the nasal sclera, simulating an esotropia. As the child grows, the nasal bridge will not be as
prominent and the folds will disappear.
Surgery on which rectus muscle is MOST likely to lead to lid malposition postoperatively:

superior rectus

lateral rectus

inferior rectus

medial rectus

Your answer was CORRECT


Explanation
Inferior rectus surgery nearly always involves manipulation of the lid retractors.

A 6-year-old child presents from school eye screening. Examination reveals:

Vision: 6/6 right; 6/36 left


Motility: left 30 PD XT, comitant
Cyclo ret: +0.50D both eyes

A year of occlusion therapy of the right eye is undertaken, with little improvement in acuity in the
left eye.

The next step in the management of this patient might be:

recession of the left lateral rectus 7 mm and resection of the left medial rectus 6 mm
prescription of +1.50D in both eyes

recession of the left lateral rectus 6 mm and resection of the left medial rectus 7 mm

base-in prism

bilateral lateral rectus recession of 7 mm

Your answer was CORRECT


Explanation
Since there is significant amblyopia, monocular surgery is preferable to minimise operative risks
on the better-seeing left eye. The guidelines for a 30 prism diopter exotropia are 6 mm of medial
rectus resection and 7 mm of lateral rectus recession.

The maximal advisable resection of the medial rectus muscle in the initial surgical management of
exotropia is:
8 mm
12 mm
10 mm
6 mm
15 mm

Your answer was CORRECT


Explanation
Maximal advisable initial surgery:
 Esotropia:
o medial recession: 6 mm
o lateral resection: 9 mm
 Exotropia:
o medial resection: 10 mm
o lateral recession: 10 mm

Which is FALSE regarding Brown's syndrome:

surgery with a superior oblique expander can be successful

both oral and local steroid administration can be beneficial

contralateral superior rectus overaction is typical

there may be an association with limbal dermoids

Your answer was CORRECT


Explanation
Limbal dermoids are associated with Duane's syndrome not Brown's. Other statements are true.

A 3-year-old has the following orthoptic report:

Vision: 6/12 both eyes


Cycloplegic refraction: +4.00D both eyes
Prism cover test (without glasses): Distance 30PD ET; Near 35 PD ET
Prism cover test (with glasses): Distance 15PD ET; Near 15PD ET
Motility: full; minimal IO overaction
Near point of accommodation: 10cm; AC/A ratio normal

What is the most likely diagnosis?

consecutive esotropia

partially accommodative esotropia


divergence-insufficiency esotropia

fully accommodative esotropia

convergence-excess esotropia
Your answer was CORRECT
Explanation
The findings are consistent with partially-accommodative esotropia.

The AC:A ratio is normal in this case so convergence-excess esotropia is ruled out. The esotropia
for distance and near are similar, so divergence insufficiency is excluded.

There is an accommodative element to the esotropia, as glasses reduce the angle but do not
eliminate it. Hence this is partially (as opposed to fully) accommodative esotropia.

Which is FALSE regarding binocular single vision?

Panum's area is broader for points in space clustered around the central portion of the
horopter and narrower in the periphery

all points lying on the horopter stimulate corresponding retinal points

for sensory fusion to exist, the two retinal images must be similar in size and shape

if corresponding retinal points in each eye have identical topographic locations relative to the
fovea, then normal retinal correspondence exists

Your answer was CORRECT


Explanation
Panum's space is broader peripherally and narrower centrally. This range describes the area of
space where an object off the horopter will still be perceived singly and with stereopsis.

Other statements above are true.

The extraocular muscle with the shortest length of active muscle belly is the:

inferior rectus

superior rectus

inferior oblique

superior oblique

Your answer was CORRECT


Explanation
The belly of the superior oblique is approximately 32 mm (the others are 37 to 40 mm)
A 7-year-old boy returns 1 day after bilateral lateral rectus recessions for intermittent exotropia.
He measures 8 PD of consecutive esotropia and has diplopia. The parents are concerned.

Which of the following management options is MOST appropriate:

suggest that reoperation may be necessary

begin convergence exercises

observe: tell the parents you are satisfied because this is the desired result day 1
prescribe prism glasses to maintain fusion

Your answer was CORRECT


Explanation
An immediate overcorrection of 8 to 12 PD is a desirable result. Often, over the first or second
postoperative week, the effect of the surgery will lessen and the eyes will straighten.

An 18-month-old child has essential esotropia of 40PD for near and distance. Cycloplegic
refraction is +1.50DS both eyes. The child is due to undergo right medial rectus recession with
lateral rectus resection.

Using the strabismus surgical tables, which of the following is the MOST appropriate target to aim
for in this case?

7 PD XT

7 PD ET

orthophoria

15 PD XT

15 PD ET

Your answer was CORRECT


Explanation
It is good practice in essential esotropia to aim for a small residual esotropia of between 7 and 10
PD post-operatively as this is well tolerated, allows good peripheral fusion and reduces the chance
of consecutive exotropia.

All are characteristic features of early-onset (infantile) exotropia EXCEPT:


patients require a neurological work-up
high incidence of dissociated vertical deviation
large angle of deviation
cycloplegic refraction between +3.00D to +5.00D
Your answer was CORRECT
Explanation
Refraction is usually normal for age. Other options are true.

Features of early-onset exotropia:


 presentation at birth or soon thereafter
 normal refraction
 large angle
 DVD
 neurological anomalies frequently present (unlike infantile eso)
 Rx: mainly surgery, e.g. LR recess/MR resect

Which one of the following indications is the weakest for intermittent exotropia surgery?

a poor recovery of fusion once tropic

an increasing ease of dissociation

an increasing degree of exodeviation in the tropia phase

a deviation of greater than 15 prism dioptres on alternate cover testing

Your answer was CORRECT


Explanation
The maintenance or preservation of good binocular function is the goal of surgery for intermittent
exotropia. The size of the deviation is of little concern if the patient is able to fuse easily.

A 6-year-old girl has limited upgaze across the horizontal plane in the right eye, while the left eye
has full range of movement. There is a chin lift.

What is the most appropriate management?

Faden procedure

superior rectus recession and inferior rectus resection

Inverse Knapp procedure

Knapp procedure
Hummelscheim procedure

Your answer was INCORRECT


Explanation
Figure: Knapp procedure

The child in the question above has right monocular elevator deficit. The cause can be
supranuclear in origin, or the result of a tight inferior rectus or hypoplastic superior rectus.

The Knapp procedure involves detachment and reinsertion of the medial and lateral recti along the
medial and lateral borders of the superior rectus.

An inverse Knapp prcedure involves transposition of medial and lateral recti to the borders of the
inferior rectus for correction of an inferior rectus palsy.

All are associated with Moebius syndrome EXCEPT:


autosomal recessive inheritance
horizontal gaze palsy
absence of the pectoralis muscle
atrophic hand
bilateral sixth nerve palsy

Your answer was CORRECT


Explanation
Figure: a child with Moebius syndrome and Poland syndrome

Moebius syndrome is characterised by:


 sporadic occurrence
 bilateral 6th and 7th nerve palsy
 gaze palsy
 9th and 12th involvement (tongue atrophy)
 limb anomalies - absent or small hand, syndactyly
 chest wall deformities (Poland syndrome) causing absent pectoralis muscle among others

A 7-year-old child with 30 PD of intermittent exotropia and 40 seconds arc stereo acuity exhibits
an A-pattern with superior oblique overaction and a small right hyperphoria in primary gaze, a left
hyperphoria in left gaze, and a larger right hyperphoria in right gaze. The right hyperphoria
significantly increases on downgaze.

What would be the most reasonable surgical approach?

Lateral rectus recessions plus superior oblique tenotomies

Lateral rectus recessions plus supraplace the lateral recti

Lateral rectus recessions plus superior oblique tuck

Lateral rectus recessions plus infraplace the lateral recti

Your answer was CORRECT


Explanation
This patient has high-grade stereoacuity and thus bifoveal fusion during his phoric phase. He also
has a significant A-pattern from bilateral superior oblique overaction. Typically, one would
operate on the superior obliques for significant overaction, but in patients with high-grade
stereopsis, superior oblique tenotomies could cause consecutive superior oblique paresis with
intractable torsional diplopia. In this case, vertical offsets of the horizontal muscles or the Wright
superior oblique tendon expander should be done. The Wright silicone expander controls the
amount of superior oblique weakening, is reversible, and alleviates the hyper in side gaze. The
surgery of choice is lateral rectus recession with infraplacement to alleviate the A-pattern.

An 18-month-old child has limited adduction of the left eye and normal abduction. There is a
small angle left exotropia and narrowing of the left palpebral fissure on dextroversion.

What is the most likely diagnosis?


congenital exotropia
Duane's type 2
Duane's type 3
Duane's type 1
Brown's syndrome

Your answer was CORRECT


Explanation
The clinical features are in keeping with Duane's type 2.

Duane’s syndrome has three variants:


 Type 1: limited abduction with or without esotropia (most common 85%)
 Type 2: limited adduction with or without exotropia
 Type 3: similar limitation of abduction and adduction and any form of strabismus

Duane's retraction syndrome (DRS):


 unilateral or bilateral abnormality of horizontal gaze
 caused by co-contraction of medial and lateral recti
 retraction of the globe on attempted adduction
 upshooting or downshooting of the globe on adduction
 the left eye is affected more frequently than the right
 females are affected more frequently than males
 Type 1: abduction limitation greater than adduction, eso (most common)
 Type 2: adduction limitation greater than abduction, exo (least common)
 Type 3: abduction and adduction limited equally
 amblyopia in 10%

Associations with Duane's retraction syndrome:


 cataracts
 iris anomalies
 Marcus Gunn jaw winking
 microphthalmos
 crocodile tears
 Goldenhar's syndrome
 maternal thalidomide
 Klippel-Feil syndrome (congenital cervical vertebral fusion, congenital high scapula, scoliosis,
spina bifida, cleft palate)

All of the following findings on examination would suggest Brown's syndrome rather than inferior
oblique palsy EXCEPT:

absence of superior oblique overaction

marked chin elevation


positive forced duction test

V-pattern

Your answer was CORRECT


Explanation
Inferior oblique palsy is an important differential to be excluded in a case of presumed Brown's
syndrome. The two conditions can be differentiated on the basis of the following:

Findings Brown's syndrome IO palsy


Hypotropia in primary position slight significant

Muscle Sequelae no ipsilateral SO overaction ipsilateral SO overaction

Alphabet Pattern V pattern A pattern

Head Posture slight marked

Forced Duction Test positive negative

A 2-year-old girl has a small angle esotropia and face turn after strabismus surgery for infantile
esotropia. Examination reveals intermittent esotropia of 15 PD with the fixing eye in adduction
even when one eye is occluded. A horizontal nystagmus is present with the fast phase to the fixing
eye and it increases when the fixing eye is abducted. Cycloplegic refraction shows + 3.00 OU.

The most likely diagnosis is:


congenital nystagmus
spasmus nutans
dissociated nystagmus
latent nystagmus with face turn to place the fixing eye in the null point

Your answer was CORRECT


Explanation
Characteristics of manifest-latent nystagmus:
 bilateral
 becomes manifest when one eye occluded, blurred or intermittently suppressed
 jerk-type nystagmus
 null point in adduction
 fast phase towards fixing eye
 face turn towards the fixing eye dampens nystagmus
 associated with interruptions to binocular development: congenital esotropia, but also
monocular congenital cataracts

Which is FALSE regarding the Worth 4-dot test when it is performed with the green lens before
the right eye and the red lens before the left eye?
it can determine the presence of both suppression and abnormal retinal correspondence
a patient who sees 5 lights has diplopia
if 2 red lights are seen this is consistent with right suppression
if 2 green lights are seen this is consistent with left suppression

Your answer was CORRECT


Explanation
3 green lights seen (not 2) is consistent with left suppression.

In a Worth 4-dot test, there are 4 lights: 1 red, 2 green and 1 white. The patient wears red-green
glasses. If the green lens is worn before the right eye and the red lens before the left eye, the
following outcomes can be interpreted:
 4 lights seen and patient is orthophoric = normal BSV
 4 lights seen and patient has manifest squint = harmonious ARC
 2 red lights seen = right suppression
 3 green lights seen = left suppression
 5 lights (2 red and 3 green) = diplopia
Note: it is possible to wear the red and green lenses before either the right or the left eyes
respectively, and the outcome of the test can be interpreted whichever convention is used. With
the red lens before the right eye, all the outcomes above are the same, except that 2 red lights
equates to left suppression and 3 green lights equates to right suppression.

On alternate-cover testing of a patient, when the left eye is covered, the right eye fixes on a
distance target. As the cover is shifted to the right eye, the left eye moves down to pick up
fixation. As the cover is moved back over the left eye, the right eye moves upward to reassume
fixation.

This set of findings is consistent with:

overaction of the inferior obliques

left hyperdeviation
overaction of the superior obliques

right hyperdeviation

dissociated vertical deviation

Your answer was CORRECT


Explanation
The left eye deviates upward under cover, and the right eye deviates downward under cover. This
is a left hyperdeviation or a right hypodeviation.

Which extra-ocular muscle is located furthest from the limbus?


lateral
inferior
superior
medial

Your answer was CORRECT


Explanation
Insertions of the recti muscles:
 Medial: 5.5 mm
 Inferior: 6.5 mm
 Lateral: 6.9mm
 Superior: 7.7 mm

A 3-month-old has a large angle esotropia and apparent cross-fixation. The LEAST likely
diagnosis is:

Mobius syndrome

congenital fibrosis syndrome

dense amblyopia

congenital esotropia

Your answer was CORRECT


Explanation
Dense amblyopia is not compatible with cross-fixation (or any fixation!)
The maximal advisable recession of the lateral rectus muscle in the initial surgical management of
exotropia is:
10 mm
6 mm
5 mm
12 mm
7 mm

Your answer was CORRECT


Explanation
Maximal advisable initial surgery:
 Esotropia:
o medial recession: 6 mm
o lateral resection: 9 mm
 Exotropia:
o medial resection: 10 mm
o lateral recession: 10 mm

A negative angle kappa simulates:


esotropia

hypotropia

exotropia

hypertropia

Your answer was CORRECT


Explanation
The normal angle kappa is positive (about 5 degrees). This means the fovea is temporal to the
optical centre of the eye and therefore the eye is slightly temporal to allow the object of regard to
fall on the fovea.

With a large positive angle kappa (e.g. dragged macula temporally) the eye appears exotropic as
the eye is tilted more temporally to focus light on the fovea.

A negative angle kappa is much less common than a positive angle and may be caused by high
myopia. With a negative angle kappa the eye appears esotropic.

Which of the following statements about amblyopia is FALSE?

a neutral density filter will generally cause a greater decrement in visual acuity than the
same filter placed over an eye with maculopathy
visual acuity is better for single-symbol than crowded targets
the incidence in the general population is approximately 2% to 3%

an RAPD cannot exclude the diagnosis

Your answer was CORRECT


Explanation
Neutral density filters will not significantly affect vision in amblyopes (and may even improve
vision), unlike patients with ocular pathology such as maculopathy or optic neuritis where vision
is adversely affected by ND filters.

Note that an RAPD can be seen in amblyopia although its presence should heighten suspicions
about an organic lesion.

An 18-month-old child has the following orthoptic findings:

Vision: 6/9 both eyes


Cover test: alternating/left esotropia
Prism cover test: BO 40 PD distance and near
Ocular movement: comitant deviation, upshoot of left eye on dextroversion
Refraction: +2.00D both eyes

What is the diagnosis?


early-onset esotropia
distance esotropia
fully accommodative esotropia
partially accommodative esotropia
paretic esotropia

Your answer was CORRECT


Explanation
The most likely diagnosis is early-onset (infantile) esotropia. The child has a low hypermetropic
correction which is normal for age, thus a fully or partially accommodative esotropia is
improbable, although we do not have a prism cover test with glasses, which would help to confirm
any accommodative element.

The following features in this case are all consistent with early-onset esotropia:
 large angle of deviation
 deviation similar for distance and for near
 refraction that is normal for age
 inferior oblique overaction

Congenital esotropia is characterised by:


 large angle esotropia, usually greater than 30 PD
 onset usually during the first few months, by definition by 6 months of age
 cross-fixation may be present
 latent nystagmus
 dissociated vertical deviation (in up to 60%-70%)
 inferior oblique overaction with V-pattern esotropia (in up to 60%-70%)
 mild hyperopia, + 1.00 to + 2.00 D

A patient with a comitant exotropia has a deviation measuring 5 prism diopters of exotropia at
distance and 15 prism diopters of exotropia at near.

His strabismus is best classified as a:


simulated divergence excess exotropia
convergence insufficiency exotropia
basic exotropia
true divergence excess exotropia
divergence excess exotropia

Your answer was CORRECT


Explanation
Types of exotropia:
 Basic exotropia: exotropia equal at distance and near
 Convergence-insufficiency exotropia: exotropia greater at near
 Divergence-excess exotropia: exotropia greater at distance than near

Note: To further classify divergence excess exotropia as true or simulated further tests are needed,
in particular a 30 minute occlusion and re-testing for near and distance. This helps to get rid of any
tonic fusional convergence for near and elicits the full exotropia at near. If this test returns with
near and distance exotropia equal, then pseudo-divergence excess is present. If true divergence
excess is present, measuring AC/A ratio is another important test to perform, as the distance:near
discrepancy may be contributed to by a high AC/A ratio, which is important to know for surgical
planning and prognosis. Specifically, patients with true divergence excess and a high AC/A ratio
are prone to overcorrection (75% overcorrection) if surgery for the full distance deviation is
performed. Patients (and parents) should be aware of the poor prognosis before surgery and of the
possible need for bifocals to decrease the high AC/A ratio postoperatively.

All are features of microtropia EXCEPT:


abnormal binocular single vision with sensory and motor fusion
foveal suppression scotoma is present in the affected eye
stereopsis is usually absent
anisometropia in the majority of cases

Your answer was CORRECT


Explanation
Stereopsis is usually reduced but not absent.

Microtropia is more prevalent in esotropia. It is classified as with or without identity. In the


former, the deviation is associated with eccentric fixation which is coincident with the angle of
abnormal retinal correspondence so no manifest deviation is detected. The visual acuity is usually
reduced and a BSV is demonstrable. Microtropia without identity is present when a very small
manifest deviation is seen on cover test. The eccentric fixation is not coincident with the angle of
abnormal retinal correspondence.

Microtropia is characterised by:


 small angle manifest deviation less than 10PD
 central suppression scotoma
 eccentric fixation
 abnormal retinal correspondence
 anisometropia common - often hypermetropia +- astigmatism
 stereopsis reduced
 amblopia
 Rx: refractive correction, occlusion for amblyopia

The superior oblique inserts on which quadrant of the globe:

posteroinferior

anteroinferior

anterosuperior

posterosuperior

Your answer was CORRECT


Explanation
Figure: Lateral orbital anatomy with the lateral rectus peeled away revealing the ciliary ganglion.
(Courtesy Patrick J Lynch, Creative Commons license 2.5)

The superior oblique inserts on the posterosuperior quadrant of the globe.

All of the following tests require glasses EXCEPT:

Titmus fly

TNO

Frisby
Worth 4-dot

Your answer was CORRECT


Explanation
Titmus fly and TNO both test stereoacuity, using Polaroid glasses and red-green glasses
respectively. The Frisby tests stereoacuity by discriminating the thickness of glass plates, with no
requirement for glasses. The Worth 4-dot tests binocular single vision, retinal correspondence and
suppression by using red-green glasses and coloured lights.

This question came in the 2014 FRCOphth.

Which of the following constitutes a violation of Hering's law?

dissociated vertical deviation

Brown's syndrome

cyclic esotropia

Duane's syndrome

inhibitional palsy of the contralateral antagonist

Your answer was CORRECT


Explanation
In DVD, the affected eye drifts up under cover. When the cover is shifted to the other eye, if
Hering's law is followed, yoke innervation should generate a contralateral hypotropia. This is not
seen in DVD for reasons that are unclear.

The surgical procedure of choice in a superior oblique paresis with excyclotorsion only and no
vertical diplopia is:

ipsilateral inferior oblique myectomy


recession of the contralateral inferior rectus

ipsilateral superior oblique tuck

anterior transposition of the anterior fibres of the superior oblique tendon


recession of the ipsilateral superior rectus muscle

Your answer was CORRECT


Explanation

This procedure, called the Harada-Ito procedure, increases the force vector for incyclotorsion by
moving the antero-lateral half of the insertion of the paretic muscle. It has no effect on vertical eye
movement or fusion.

Which extraocular muscle is inserted nearest to the cornea:


lateral rectus
medial rectus
superior rectus
inferior rectus

Your answer was CORRECT


Explanation
The insertions of the recti muscles from the limbus are as follows:
 medial rectus 5.6mm
 inferior rectus 6.6mm
 lateral rectus 7.0mm
 superior rectus 7.8mm

A 5-year-old child with an amblyopic left eye is best treated with:


full-time patching of the right eye for 6 weeks

full-time patching of the right eye for 3 weeks

part-time patching of the left eye for 4 weeks

optical degradation of the left eye for 6 weeks

Your answer was CORRECT


Explanation
The goal of amblyopia treatment is to force the use of the poorer seeing (amblyopic) eye by
limiting the use of the better eye with occlusion or image degradation. The duration of full-time
occlusion must not exceed 1 week per year of age in order to prevent occlusion (deprivation)
amblyopia.

Which is the largest extraocular muscle:

medial rectus

inferior rectus

superior rectus

lateral rectus

Your answer was CORRECT


Explanation
The medial rectus is the largest extraocular muscle.

A 27-year-old man with moderate hyperopia presents for routine examination. There is a 10-
prism-diopter alternating esotropia at distance. While reading through his distance correction at
20cm, there is a 35-prism-diopter esotropia. His interpupillary distance is 60 mm, and his near
deviation increases to 50 prism diopters when he views an acuity target through a -1.00D sphere
over each eye.

By the gradient method, his AC/A measures:


50:1
25:1
5:1
11:1
15:1

Your answer was CORRECT


Explanation
By the gradient method, accommodative convergence to accommodation ratio (AC/A) equals the
difference in the deviation induced by a lens divided by the specific accommodative gradient (of
an extra lens over the distance correction). Minus lenses stimulate accommodation, whereas plus
lenses blunt it. In this example: (50 - 35)/1 = 15:1.

AC/A ratio can be formerly calculated by:


 Heterophoria method:
[(near deviation – distance deviation)/accommodative demand]+interpupillary distance in cm
 Gradient method:
difference in deviation induced by a lens/accommodative gradient
 Clinical method:
if an accommodative esotropia is 10PD greater at near than distance AC/A ratio is said to be
clinically high
Note: A normal AC/A ratio is between 3 to 5.

A 2-year-old child with esotropia since birth has right hypertropia on left gaze and left hypertropia
on right gaze. The incomitance of this deviation is most likely secondary to a muscle that:
passes below sclera and above an adjacent muscle
passes between the sclera and a rectus muscle
elevates, intorts, and adducts
has its insertion near the macula

Your answer was CORRECT


Explanation
The above case is classic for congenital ET with bilateral inferior oblique overaction. The inferior
oblique inserts near the macula.

Congenital esotropia is characterised by:


 large angle esotropia, usually greater than 30 PD
 onset usually during the first few months, by definition by 6 months of age
 cross-fixation may be present
 latent nystagmus
 dissociated vertical deviation (in up to 60%-70%)
 inferior oblique overaction with V-pattern esotropia (in up to 60%-70%)
 mild hyperopia, + 1.00 to + 2.00 D

Which is FALSE regarding congenital strabismus?

congenital exotropia requires brain imaging

congenital exotropia is less common than congenital esotropia

congenital esotropia is typically secondary to high hypermetropia

congenital esotropia is rarely associated with intracranial pathology

Your answer was CORRECT


Explanation
Congenital esotropia usually involves moderate hypermetropia (+1.00 to +2.00D).
Accommodative esotropia develops later at ages 2 to 3. Congenital exotropia is rarer than
congenital esotropia and it is much more likely to be associated with neurological abnormalities.
Brain imaging should be considered.

Which is FALSE regarding Duane's syndrome:


inferior oblique overaction is common
left eye is more commonly involved than the right eye
more common in females
epibulbar dermoids, deafness, cafe-au-lait spots and spine anomalies are known associations
limitation of abduction is more common than adduction

Your answer was CORRECT


Explanation
Duane's syndrome is often associated with upshoot or downshoot in adduction. However, this
phenomenon is not the result of oblique muscle overaction; instead, it is the result of co-
contraction of the medial and lateral rectus muscles (the aetiology of Duane's) with slippage of the
globe.

Duane's retraction syndrome (DRS):


 unilateral or bilateral abnormality of horizontal gaze
 caused by co-contraction of medial and lateral recti
 retraction of the globe on attempted adduction
 upshooting or downshooting of the globe on adduction
 the left eye is affected more frequently than the right
 females are affected more frequently than males
 Type 1: abduction limitation greater than adduction, eso (most common)
 Type 2: adduction limitation greater than abduction, exo (least common)
 Type 3: abduction and adduction limited equally
 amblyopia in 10%
Associations with Duane's retraction syndrome:
 cataracts
 iris anomalies
 Marcus Gunn jaw winking
 microphthalmos
 crocodile tears
 Goldenhar's syndrome
 maternal thalidomide
 Klippel-Feil syndrome (congenital cervical vertebral fusion, congenital high scapula, scoliosis,
spina bifida, cleft palate)

Each of the following may be associated with Duane's retraction syndrome EXCEPT:
trabecular dysgenesis
Marcus Gunn jaw winking
Goldenhar's syndrome
thalidomide

Your answer was CORRECT


Explanation
Duane's retraction syndrome (DRS):
 unilateral or bilateral abnormality of horizontal gaze
 caused by co-contraction of medial and lateral recti
 retraction of the globe on attempted adduction
 upshooting or downshooting of the globe on adduction
 the left eye is affected more frequently than the right
 females are affected more frequently than males
 Type 1: abduction limitation greater than adduction, eso (most common)
 Type 2: adduction limitation greater than abduction, exo (least common)
 Type 3: abduction and adduction limited equally
 amblyopia in 10%
Associations with Duane's retraction syndrome:
 cataracts
 iris anomalies
 Marcus Gunn jaw winking
 microphthalmos
 crocodile tears
 Goldenhar's syndrome
 maternal thalidomide
 Klippel-Feil syndrome

An asymptomatic, 4-year-old child with a moderate-angle esotropia is noted to have a left


hypertropia on right gaze and a right hypertropia on left gaze. When fixing with the left eye in
right gaze, there is a right hypotropia, and when fixing with the right eye in left gaze, there is a left
hypotropia.

The most likely clinical diagnosis is:

left hypotropia

esotropia associated with overaction of the inferior oblique muscles

bilateral trochlear palsy

esotropia with dissociated vertical deviation

right hypotropia

Your answer was CORRECT


Explanation
These features are common in infantile esotropia. There are two potential explanations for the
vertical deviation in lateral gaze. The first is overaction of the inferior obliques. In this condition,
on lateral gaze, there is hypotropia of the abducted, non-fixing eye. In dissociated vertical
deviation (DVD), in lateral gaze there is no associated hypotropia, in violation of Hering's law. In
fact, if the DVD is bilateral, there may be hyperdeviation of the abducted, nonfixing eye. The
distinction between the two is important for surgical planning.

Note, the finding of right hypertropia in left gaze and left hypertropia in right gaze can also occur
in bilateral trochlear palsy. However, this often occurs after trauma and the patient is very
symptomatic of torsional diplopia (not consistent with this case). There is often a chin-down
posture and head tilt is positive on tilting either side. While a V-pattern esotropia can occur with
bilateral SO palsy, the esotropia is a much smaller angle than occurs in infantile strabismus with
bilateral IO overaction.

Which is FALSE regarding inferior oblique overaction:

may cause hypertropia in adduction or abduction

may be corrected by anteriorisation of the tendon to Marshall Parks point 3mm lateral to lateral
border of inferior rectus insertion and 1mm behind

typically associated with a V-pattern deviation

may be associated with large-angle esotropia

Your answer was CORRECT


Explanation
Inferior oblique overaction causes hypertropia in adduction, but not in abduction. This is one of
the characteristics used to differentiate IO overaction from DVD. In DVD elevation can occur in
primary position, abduction and adduction.

Part 2 Premium – Tutor Mode


Question 2 of 10
Score: 100 %
The most common complication of botulinum injections in squint surgery is:

vertical strabismus

ptosis
retrobulbar haemorrhage

Adie's pupil

perforation of the globe


Your answer was CORRECT
Explanation
Ptosis is the most common and is seen slightly more frequently in children. Secondary vertical
strabismus is the second most common complication.

Which is TRUE of the Parks' 3-step test?

most applicable when a single muscle is involved

distinguishes a palsy from an overactive muscle

separates congenital from acquired disorders

able to differentiate between restrictive and paralytic palsies

Your answer was CORRECT


Explanation
The Parks' three-step test is very useful in determining the specific pattern of deviations in single
muscle palsies or overactions. The results are inconsistent or unreliable when more than one
muscle is affected. The test is not able to distinguish between restrictive or paralytic palsies and
congenital or acquired conditions. As with all tests, it is an adjunct to other clinical signs and
symptoms in determining the involved muscle and whether it is palsied or overacting.

A 38-year-old man presents complaining of diplopia and difficulty descending stairs since an
automobile accident 1 week earlier. The patient has a left head tilt and his general practioner
concludes that he must have a right superior oblique paresis.

Which one of the following findings could not possibly be present if the examiner is correct?
left hypertropia in right gaze

right hypertropia aggravated by right gaze


V-pattern esotropia

right hypertropia in primary position

right hypertropia aggravated by left gaze

Your answer was CORRECT


Explanation
The finding of a left hypertropia in right gaze indicates that more than a simple right fourth palsy
is present but does not eliminate this possibility because the left hypertropia is seen only in right
gaze (indicating probable bilateral superior oblique palsy). A right hypertropia in primary gaze
that is worsened with left gaze is classic for right SO palsy. The V-pattern esotropia again
suggests bilateral superior oblique palsy. However, a right hypertropia worse in right gaze, by the
three-step test, maps to the right inferior rectus or the left inferior oblique and is therefore
inconsistent with the diagnosis.

All are features of amblyopia EXCEPT:


visual acuity is reduced with crowded letters
can be expected when there is astigmatism of 0.75D or more
more common in hypermetropic anisometropia than an equivalent degree of myopic
anisometropia
more common in high hypermetropia than in an equivalent myopic prescription

Your answer was CORRECT


Explanation
Refractive errors causing amblyopia:
 hyperopia over 4-5 D
 myopia over 6-7 D
 anisometropia: difference over 1.50 for hyperopes or 3.00D for myopes
 meridional: cyclinder over 1.50D
Note: amblyopia is less likely in myopia compared to an equivalent degree of hyperopia because
children with myopia can still focus clearly for near objects.

Superior oblique overaction is MOST likely to be encountered in:

A-pattern esotropia

A-pattern exotropia

V-pattern exotropia

V-pattern esotropia

Your answer was CORRECT


Explanation
The superior oblique muscles are abductors, particularly in downgaze. Thus, overaction will result
in overabduction (exotropia) in downgaze (A pattern).

All of the following are features consistent with double elevator palsy EXCEPT:

ptosis

chin-down head position

poor Bell phenomenon on the side of the palsy

ipsilateral hypotropia with large secondary deviation


forced ductions indicating inferior rectus restriction

Your answer was CORRECT


Explanation

Figure: girl with left monocular double elevator palsy on attempted upgaze.

Double elevator palsy may be caused by elevator weakness or restriction of the depressors. Ptosis,
hypotropia, and poor elevation in any direction are characteristic. A subset will have positive
forced ductions for the inferior rectus. Head position is generally an automatic compensation for
the hypotropia, with the chin up.

A Maddox rod test is performed on a patient. With the rod held before the right eye, he sees a red
line above a point of light.

What is the diagnosis:


right esophoria
orthophoria
right hypophoria
right hyperphoria
right exophoria

Your answer was CORRECT


Explanation
Figure: The patients view of a Maddox rod before the right eye.

1. Horizontal orthophoria
2. Exophoria (the patient has crossed diplopia and this indicates an exophoria (X'ed diplopia =
eXo)
3. Esophoria
4. Vertical orthophoria
5. Right hyperphoria
6. Right hypophoria (=left hyperphoria)

Maddox rod test interpretation:


 Line above point: covered eye is hypo-deviated
 Line below point: covered eye is hyper-deviated
 Line temporal: covered eye is eso-deviated
 Line nasal: covered eye is exo-deviated

Regarding squint surgery all of the following are true under most circumstances EXCEPT:

recession of the lateral rectus is more effective than recession of the medial rectus
resection has a greater effect than an equivalent amount of recession

deviation that is greater for near than distance is better corrected by medial rectus surgery than
lateral rectus

adjustable surgery is preferred to fixed sutures in adult re-operations on horizontal recti

Your answer was CORRECT


Explanation
Recession of the medial rectus has more effect than an equivalent recession of the lateral rectus.
All other statements are true.

Bilateral superior oblique tenotomies for correction of an A-pattern exotropia, can be expected to
cause how much of an esodeviation in downgaze?

50 prism diopters

5 prism diopters

40 prism diopters

10 prism diopters

20 prism diopters

Your answer was CORRECT


Explanation
A bilateral superior oblique tenotomy causes approximately 40 prism dioptres of esodeviation in
downgaze (to correct an A pattern). The procedure has little eso effect in primary gaze and none in
upgaze.

A bilateral inferior oblique myectomy produces 15 to 25 prism diopters of esodeviation in upgaze


(to correct a V pattern).

Which is FALSE regarding manifest-latent nystagmus?


may become manifest in the setting of monocular visual loss
fast phase is toward the uncovered eye
associated with infantile esotropia
face turn towards the covered eye dampens nystagmus

Your answer was CORRECT


Explanation
Characteristics of manifest-latent nystagmus:
 bilateral
 becomes manifest and/or amplitude increases when one eye occluded, blurred or intermittently
suppressed
 jerk-type nystagmus
 null point in adduction
 fast phase towards fixing eye
 face turn towards the fixing eye dampens nystagmus
 associated with interruptions to binocular development: congenital esotropia, but also
monocular congenital cataracts

Note: latent nystagmus is absent when both eyes are open and is only present when the light
stimulus to one eye is reduced. Manifest-latent nystagmus is present with both eyes open, but the
amplitude is increased when the light stimulus to the fixing eye is reduced. Latent nystagmus and
manifest-latent nystagmus are the same condition and any difference between them can be
considered quantitative. Reference: Diagnosis and Management of Ocular Motility Disorders by
Ansons and Davis, 2008.

Clinical features characteristically associated with intermittent exotropia include all EXCEPT:

variable angle of deviation

normal stereoacuity

reflex closure of one eye in bright light

high AC/A ratio

amblyopia

Your answer was CORRECT


Explanation
Amblyopia is not a feature of intermittent XT because the deviation is intermittent. Most patients
with truly intermittent exotropia have excellent stereoacuity. High accommodative AC/A ratios
may develop as a fusional mechanism for near work. The deviation in intermittent exotropia is
highly variable and sensitive to external stimuli (stress, alcohol, tiredness). Bright light typically
causes reflex closure of the deviating eye.

Parents bring their 3-year-old boy for examination having noticed a squint. It has been present
throughout the day since he was 2 years old. A brief inspection of the child shows an obvious,
right, constant moderate-angle esotropia.

Which of the following is FALSE?

this disorder may have a family history

an accommodative component is unlikely

stereopsis testing must be negative

amblyopia is likely

the deviation could have been originally intermittent

Your answer was CORRECT


Explanation
Amblyopia is certainly possible given the constant esotropia. A large-angle turn might provide
cross-fixation, but amblyopia must be ruled out. Given the age of onset, an accommodative
component is likely, so an originally intermittent turn is also possible. No patient with manifest
strabismus has any stereopsis whatsoever. Certainly, a family history of esotropia of any
mechanism is possible.

The maximal advisable resection of the lateral rectus muscle in the initial surgical management of
esotropia is:
9 mm
10 mm
6 mm
7 mm
8 mm

Your answer was CORRECT


Explanation
Maximal advisable initial surgery:
 Esotropia:
o medial recession: 6 mm
o lateral resection: 9 mm
 Exotropia:
o medial resection: 10 mm
o lateral recession: 10 mm

Broad nasal bridges with abnormally large angle kappa may lead to an error in the diagnosis of
strabismus with which of the following methods?
Hirschberg testing
Maddox rod testing

cover-uncover testing

alternate-cover tests

Your answer was CORRECT


Explanation
The first three tests utilize actual fixation behavior and will not fall victim to large angle kappa or
facial anomalies. The Krimsky and Hirschberg methods use apparent eccentricity of pupillary
reflexes and may be misguided by these factors.

Which is the only extraocular muscle not to originate at the orbital apex:

the inferior oblique

the inferior rectus

the superior rectus

the superior oblique

Your answer was CORRECT


Explanation
The inferior oblique is the only extrocular muscle to originate outside the orbital apex. It
originates behind the orbital margin lateral to the nasolacrimal duct and passes between the eye
and the lateral rectus.

During routine examination, an alternate-cover test reveals outward fixation shifts of each eye as
the cover is moved. The cover-uncover test reveals no shift of either eye as the cover is placed
over either eye.

The correct description of the patient's motility status would be:

orthotropic, esophoric
orthophoric, esotropic

orthotropic, exophoric

orthophoric, orthotropic

Your answer was CORRECT


Explanation
There is no manifest deviation (tropia, cover-uncover test) but there is a latent esodeviation
(phoria, alternate-cover test).

If a patient with untreated congenital esotropia is tested with the Hess chart, when the glasses are
reversed and the test is repeated, which one of the following statements is true?
the position of the lights on the chart will reverse, and the distance will remain the same

the test cannot be performed

the position of the lights on the chart will not reverse, and the distance will increase

the position of the lights on the screen will reverse, and the distance between them will increase

the position of the lights on the chart will remain the same

Your answer was CORRECT


Explanation
Because of suppression, the Hess test is not useful for congenital strabismus, but only for acquired
cases. In this case, the patient will see only one light.

The most appropriate treatment for latent nystagmus with a small intermittent esotropia measuring
15PD and a cycloplegic refraction of +3.00D OU is:
Botox injection of both medial rectus muscles
prescribe full hyperopic correction + 3.00 OU
bilateral medial rectus recessions
convergence exercises

Your answer was CORRECT


Explanation
Latent nystagmus can be reduced by correcting any residual esotropia and improving binocular
fusion. The best way to correct the small esotropia in this case is by giving the hyperopic
correction. Improving binocular fusion will reduce the latent nystagmus, thus reducing the face
turn.

Note: in any case with an esodeviation and nystagmus, the differential includes infantile eso with
manifest-latent nystagmus (as in this case) OR congenital nystagmus with nystagmus blockage
syndrome.

Nystagmus blockage syndrome is characterised by:


 congenital nystagmus (not manifest-latent nystagmus)
 accommodative convergence to dampen nystagmus for near
 variable-angle esodeviation for near only
 esodeviation increases with a base out prism
 pupillary miosis with esotropia (shows accommodative element)
 inferior oblique overaction and DVD not as common as infantile esotropia

Which operation is MOST appropriate for a patient with an A pattern deviation?

medial and lateral rectus transposed inferiorly

medial and lateral rectus transposed superiorly

medial rectus transposed superiorly and lateral rectus transposed inferiorly


medial rectus transposed inferiorly and lateral rectus transposed superiorly

Your answer was CORRECT


Explanation
The MALE acronym is useful to remember where the recti muscles are transposed to in A and V
pattern deviations.
Medial recti = transposed to Apex
Lateral recti = transposed to Ends

This question appeared in the 2014 FRCOphth Part 2.

Which of the following clinical findings implies the presence of a bilateral rather than a unilateral
superior oblique paresis?
aggravation of diplopia with right and left head tilt
head tilt
A-pattern esotropia
symptomatic excyclotorsion

Your answer was CORRECT


Explanation
Head tilt occurs in unilateral SO palsy; but not in bilateral SO palsy, where patients prefer a chin-
down posture. Excyclotorsion occurs in both unilateral and bilateral palsy. V-pattern (not A-
pattern) esotropia is consistent with bilateral SO palsy.

Bilateral superior oblique is suggested by the following:


 excyclodeviation over 10 PD
 minimal head tilt
 chin-down head position
 alternating hypertropia on lateral gaze
 positive head-tilt bilaterally
 V-pattern esotropia

A patient presents with:

primary position: 20 PD ET
downgaze: 35 PD ET
upgaze: 15 PD ET
motility: inferior oblique overaction bilaterally

Appropriate surgical intervention might include each of the following steps EXCEPT:

recession of the ipsilateral medial rectus muscle

upward transposition of the lateral rectus and downward transposition of the medial rectus
muscles ipsilaterally
resection of the ipsilateral lateral rectus muscle

bilateral inferior oblique myectomies

Your answer was CORRECT


Explanation
In V-pattern esotropia with overacting inferior obliques, inferior oblique myectomy will cause 15
to 25 prism diopters of esoshift in upgaze. This only makes the deviation more comitant. Thus, the
deviation in primary gaze must be addressed with appropriate medial rectus recession and lateral
rectus resection. Because oblique surgery is indicated, rectus transpositions are not.

All of the following are characteristic findings in infantile esotropia EXCEPT:


latent nystagmus
overaction of the inferior obliques
dissociated vertical divergence
high accommodative convergence to accommodation ratio

Your answer was CORRECT


Explanation
A high AC/A ratio may be seen but is not typical.

Infantile esotropia is characterised by:


 large angle esotropia, usually greater than 30 PD
 onset usually during the first few months, by definition by 6 months of age
 cross-fixation may be present
 latent nystagmus
 dissociated vertical deviation (in up to 60%-70%)
 inferior oblique overaction with V-pattern esotropia (in up to 60%-70%)
 mild hyperopia, + 1.00 to + 2.00 D

A 4 PD base out prism test is most useful for diagnosing:

abnormal retinal correspondence

functional visual loss

binocular single vision

stereoacuity

central suppression scotoma

Your answer was CORRECT


Explanation
A 4 PD base out test is most useful for diagnosing a central suppression scotoma as seen in
microtropias.

When the prism is placed before the suppression eye, there is no movement detected.

When the prism is placed before the fellow eye, there is movement of the fellow eye (and
conjugate movement of the suppression eye) towards the apex, which is normal behaviour.
However, there is no re-fixation movement of the affected eye as expected in normal
circumstances.

A 57-year-old woman presents to A&E with recent-onset right esotropia. A red glass is placed in
front of her left eye and the patient is asked to fixate on a distant point target. She reports the
white light to be to the right of the red light. These images are superimposed with a 10-prism-
diopter prism placed base-out over the left eye. Simultaneous prism cover test with a distance
target and no red glass reveals a 20-prism-diopter right esotropia.

These results indicate:


central suppression

visual confusion

harmonious anomalous retinal correspondence

unharmonious anomalous retinal correspondence


normal retinal correspondence

Your answer was CORRECT


Explanation
If the angle of the tropia (measured by simultaenous prism cover test) and the angle of subjective
image separation are equal, there is normal retinal correspondence (NRC). If there is a
discrepancy, then there is abnormal retinal correspondence (ARC). If ARC is harmonious, there
will be fusion of images (i.e. no diplopia; subjective angle of image separation zero) with no prism
in place. In unharmonious ARC, a prism is required to fuse images (eliminate diplopia), but the
size of this prism does not equal the size of the tropia.
Measurements for ARC/NRC can be obtained with an amblyoscope, to project similar images
onto each fovea. The angle of deviation between the two targets matches the tropia angle in NRC.
If the angle of deviation is less than the tropia, ARC is unharmonious. If the angle is 0 in the
setting of a tropia, ARC is harmonious.

Which of the following statements about A and V patterns of horizontal strabismus is TRUE?
these forms of noncomitance are seen in less than 5% of horizontal strabismus
A-patterns must measure at least 15-prism diopters between upgaze and downgaze to be
considered significant
all the extraocular muscles in varying combinations have been implicated as responsible for
these patterns
V patterns must measure at least 10 prism diopters between upgaze and downgaze to be
considered significant

Your answer was CORRECT


Explanation
Alphabet patterns:
 A pattern: significant if 10 prism diopters
 V pattern: significant if 15 prism diopters

Approximately 15% of horizontal strabismus cases have a significant A or V component.

A 25-year-old develops right abducens palsy following a road traffic accident with head trauma.
He has a right esotropia and is tested with the Lancaster red-green test. He wears the goggles with
the red glass over his right eye and the green glass over his left. An examiner holds the green light
central on the chart and gives the patient the red light. The patient is then instructed to
superimpose his red light on the examiner's green light.

To the examiner:
the red light will appear above the green light

the red light will appear to the right of the green light

the lights will be superimposed

the red light will appear below the green light

the red light will appear to the left of the green light

Your answer was CORRECT


Explanation
In the Lancaster red-green test, the fovea of each eye is isolated with duochrome glasses. The eye
under green glass (left) will not see the red light and the eye under the red glass (right) will not see
the green light. By holding the green light centrally, the examiner holds the patient's left fovea
centrally. Assuming normal retinal correspondence (NRC), the patient will direct the red light in
space along his right visual axis to place the red light on his right fovea and superimpose the
foveal images. The patient's right visual axis (in esotropia) crosses his left visual axis, so the right
foveal image will fall to the left of the left foveal image, as seen by the examiner. To the patient,
the images appear superimposed (assuming NRC).
Note: The Lancaster red-green test is very similar to the Hess. The main difference between the
Hess and Lancaster tests is that the Lancaster projects a line whereas the Hess pointers project a
simple dot. Lancaster line generators permit determination of the orientation of the patient's
vision. Orientation is important in understanding any cyclo-deviation resulting from ocular muscle
imbalance. The Hess test may be more useful in understanding full visual field and any
aniseikonic affects. Further, the Hess test may be more simple for children to understand thus
yielding more reliable results in paediatric situations.

Strabismus surgery for patients with Graves' ophthalmopathy is generally performed:

before orbital decompression

before tarsorrhaphy

before eyelid surgery

before orbital radiation

Your answer was CORRECT


Explanation
Muscle surgery, particularly vertical muscle surgery often indicated in Graves' disease, can affect
the position of the eyelids. As a result, it is generally wise to perform any strabismus surgery
before eyelid repositioning.

However, strabismus surgery should be performed after treatments for orbital disease have been
completed including orbital decompression and radiation, as these treatments can alter the
strabismic effects of TED and change decisions for surgery.

A 4-year-old boy presents with a decompensating exophoria of 30PD for distance and 25PD for
near. The exotropia measures 45PD in upgaze and 20PD in downgaze. He has 1+ overacting
inferior obliques. He has no evidence of amblyopia.

The most reasonable surgical approach is:

resection of both medial recti with supraplacement

recession of both lateral recti with inferior oblique weakening

recession with supraplacement of lateral rectus and resection with infraplacement of medial rectus
recession of both lateral recti with supraplacement

Your answer was CORRECT


Explanation
This patient exhibits a significant V-pattern without significantly overacting inferior obliques. In
general, inferior oblique surgery is indicated for overaction of 2+ or more. Without significant
oblique overaction, offsetting the horizontal muscles can correct as much as 30 PD of an A- or V-
pattern. Horizontal muscle offset changes the vector of forces. The medial recti are moved toward
the apex of the A- or V-pattern while the lateral recti are moved in the opposite direction.

Useful pnemonic for horizontal transpositions: MALE (medial to apex, lateral to the ends)

A recess-resect procedure is useful in cases in which surgery must be limited to one eye or when
an incomitant deviation exists. Horizontal offset may be performed in conjunction with the recess-
resect, but it may not correct large A- or V-patterns. In this case, the procedure of choice would be
to recess the lateral recti for the appropriate deviation in primary position and offset the lateral
recti superiorly.

A 43-year-old woman presents with insidious onset of diplopia. On alternate cover testing, the
patient has a right hypertropia, worse on right head tilt and left gaze.

A palsy of which muscle might cause her symptoms?

left superior rectus

right superior oblique

right inferior rectus

left inferior oblique

Your answer was CORRECT


Explanation
This is the Parks' three-step test.

The Parks' three-step test helps to identify a vertical muscle palsy:

1. Determine which eye is hypertropic. This tells you that the involved muscle is one of the two
depressors in the hypertropic eye, or one of the two elevators in the hypotropic eye. With this first
step complete, choices are narrowed from eight muscles down to four muscles.

2. Decide in which gaze the hypertropia is worse. By using the field of action of the four vertically
acting muscles, the choices can always be narrowed down to two. In this example, the right
hypertropia, which is worse on left gaze, indicates either the right superior oblique (the depressor
of the right eye in left gaze) or the left superior rectus (the elevator of the left eye in left gaze)

3.Determine if right or left head tilt worsens the hypertropia. If the deviation is worse when the
head is tilted towards the hyper-tropic eye, then the defect is in an oblique muscle; if the deviation
is worse when the head is tilted towards the hypo-tropic eye, then the defect is in a rectus muscle.

Which is TRUE regarding dissociated vertical deviation (DVD)?


usually a unilateral condition
the deviated eye intorts as it elevates
it violates Hering's law
it is rare in patients with congenital esotropia

Your answer was CORRECT


Explanation
Dissociated vertical deviation:
 present in 60% to 80% of patients with congenital esotropia
 usually bilateral and asymmetric
 aetiology unknown
 probably due to early disruption of binocular development
 high-grade stereopsis and bifoveal fixation are not seen
 during visual inattention, the non-fixating eye slowly drifts up, extorts, and abducts without a
corresponding hypotropia of the fellow eye on alternate cover testing
 thus, it does not obey Hering's law
Note: DVD can simulate inferior oblique overaction (IOOA) in side gaze when the nose acts as an
occluder. However, the hyperdeviation in DVD is of the same amount in adduction, abduction,
and primary position. This is in contrast to IOOA, in which the hyperdeviation is greatest in its
field of action.

Which is TRUE regarding dissociated vertical deviation (DVD)?


usually a unilateral condition
the deviated eye intorts as it elevates
it violates Hering's law
it is rare in patients with congenital esotropia

Your answer was CORRECT


Explanation
Dissociated vertical deviation:
 present in 60% to 80% of patients with congenital esotropia
 usually bilateral and asymmetric
 aetiology unknown
 probably due to early disruption of binocular development
 high-grade stereopsis and bifoveal fixation are not seen
 during visual inattention, the non-fixating eye slowly drifts up, extorts, and abducts without a
corresponding hypotropia of the fellow eye on alternate cover testing
 thus, it does not obey Hering's law
Note: DVD can simulate inferior oblique overaction (IOOA) in side gaze when the nose acts as an
occluder. However, the hyperdeviation in DVD is of the same amount in adduction, abduction,
and primary position. This is in contrast to IOOA, in which the hyperdeviation is greatest in its
field of action.

A patient undergoes the Worth four-dot test and reports suppression with distance testing, but
fusion on near testing.

How would you interpret this result?

peripheral fusion but no central fusion

alternating squint

central fusion but no peripheral fusion

retinal rivalry

alternating suppression

Your answer was CORRECT


Explanation
When the Worth 4-dot is performed at distance, central retinal elements are stimulated, and central
fusion is evaluated. When performed at near, more peripheral fusion processes are tested. The test
is not capable of detecting small suppression scotomata (i.e. results of near Worth four-dot testing
may be normal in the setting of a central scotoma).

Which one of the following statements regarding Duane's retraction syndrome is TRUE?
the lid fissure narrowing is secondary to abnormal innervation of the levator muscle
a Faden procedure may help reduce upshoot of the affected eye on adduction
the strabismus is comitant
amblyopia is present in 50%

Your answer was CORRECT


Explanation
A Faden procedure (posterior fixation of the horizontal rectus muscles near the equator) may
reduce the upshoot of the affected eye on adduction, as it stops vertical slippage of the lateral
rectus muscle.

Duane's retraction syndrome (DRS):


 unilateral or bilateral abnormality of horizontal gaze
 caused by co-contraction of medial and lateral rectus muscles
 retraction of the globe on attempted adduction
 upshooting or downshooting of the globe on adduction
 the left eye is affected more frequently than the right
 females are affected more frequently than males
 Type 1: abduction limitation greater than adduction, eso (most common)
 Type 2: adduction limitation greater than abduction, exo (least common)
 Type 3: abduction and adduction limited equally
 amblyopia in 10%
Associations with Duane's retraction syndrome:
 cataracts
 iris anomalies
 Marcus Gunn jaw winking
 microphthalmos
 crocodile tears
 Goldenhar's syndrome
 maternal thalidomide
 Klippel-Feil syndrome

A 9-year-old boy presents with a 1-year history of intermittent squint. On examination, he has an
exotropia measuring 10 prism dioptres XT for near and 20 prism dioptres XT for distance.
Cycloplegic refraction reveals a +4.00D correction in both eyes. You prescribe appropriate
glasses.

The glasses are likely to cause:

decrease in the angle for distance and increase in the angle for near

increase in the angle for distance and decrease in the angle for near

no change to his exotropia

increase in the angle for distance and near


decrease in the angle for distance and near

Your answer was CORRECT


Explanation
By treating his hypermetropia, this will reduce his accommodative drive for convergence and
cause an increase in the angle for distance and for near.

This question came in the FRCS (Glasgow) Part 2 exam in October 2014.

A 3-year-old girl attends clinic with a squint. It has been present throughout the day since she was
2 years old. A brief inspection of the child shows an obvious, right, constant, moderate-angle
esotropia. A cycloplegic refraction is performed and reveals +8.50D in both eyes.

The initial step in management of the patient in question must be:

full correction of the cycloplegic refractive error


penalization with atropine bilaterally
listing for bimedial recession

alternate occlusion therapy if the visual acuity is normal bilaterally

bifocals

Your answer was CORRECT


Explanation
This patient probably has refractive accommodative esotropia. In all cases of accommodative
esotropia, full hyperopic correction is warranted immediately. Penalization with atropine may be
useful in cases of non-compliance with spectacles and/or patching therapy. Bifocals may be of
value with high AC/A ratios with a residual turn at near after full distance correction and an
acceptable distance alignment.

A 34-year-old man complains of vertical diplopia since a bicycle fall 6 weeks ago. On entering the
examination room, you notice the gentleman has a head tilt and face-turn to the right and chin
down positioning.

What is the most likely diagnosis:


left SO
left brown
right sixth
right SO
right brown

Your answer was CORRECT


Explanation
Common head postures:
 Superior oblique palsy: head-turn and head-tilt to the opposite side, chin down
 Brown's syndrome: chin up
 Lateral rectus palsy: head turn towards the affected side

This question came in the FRCS (Glasgow) October 2014 exam.

The mechanism for botulinum toxin is:

direct blockage of muscle cell membrane calcium channels

direct blockage of postsynaptic acetylcholine receptors

inhibition of the formation of actin-myosin complexes

sarcoplasmic capture of extracellular calcium

inhibition of release of acetylcholine from presynaptic nerve terminals


Your answer was CORRECT
Explanation
Botulinum toxin inhibits the release of ACh from presynaptic nerve terminals.

Which of the following statements about the treatment of accommodative esotropia is FALSE?

delay in refractive correction of an accommodative esotropia increases the probability of a


permanent residual esotropia after full correction is given

surgical realignment should be attempted before considering occlusion therapy

miotics can improve esotropias with a high AC/A ratio

surgical realignment resulting in a residual esotropia of less than 10 PD can permit the
development of peripheral fusion

Your answer was CORRECT


Explanation
Surgical results are much more stable and predictable in the setting of maximal visual acuity (after
occlusion therapy). Other statements are true.

A 5-year-old girl has limited elevation on adduction of the right eye. There is no superior oblique
overaction.

All of the following are true of this condition EXCEPT:


an inelastic superior oblique muscle tendon complex
down shoot in adduction
an A-pattern is common
forced ductions in this case will be positive

Your answer was CORRECT


Explanation
Brown's syndrome is characterised by:
 congenital or acquired aetiology
 inelastic superior oblique muscle tendon complex
 restriction of passive or active elevation in adduction
 downshoot of the eye on adduction (but no superior oblique overaction)
 no superior oblique overaction
 V-pattern is common

Brown's can be differentiated from inferior oblique palsy by:


 positive forced duction test in Brown's, negative in IO palsy
 superior oblique overaction is uncommon in Brown's but typical of IO palsy

1 dioptre of accommodation usually causes:

3-5 prism dioptres of convergence

1 prism dioptre of covergence

10 prism dioptres of convergence

10 degrees of convergence

Your answer was CORRECT


Explanation
The normal AC:A ratio is 3-5, meaning that 1 dioptre of accommodation causes 3-5 prism dioptres
of convergence.

A young girl has short stature, diabetes, deafness, pigmentary retinopathy and chronic progressive
external ophthalmoplegia.

What is the most likely histological finding?


Schaumann
Langhan
Touton cells
Cowdry type A bodies
ragged red fibres in skeletal muscle

Your answer was CORRECT


Explanation
This patient has a history suggestive of Kearn-Sayre syndrome. Ragged red fibres in skeletal
muscle, due to accumulation of mitochondria are found histologically in Kearn-Sayre.

Kearns-Sayre syndrome is characterised by:


 mitochondrial inheritence
 ragged red fibres on muscle biopsy
 presentation in 1st or 2nd decades
 ptosis
 external ophthalmoplegia
 cardiac conduction defects
 deafness
 diabetes
 short stature
 mild pigmentary retinopathy
 A 3-year-old child presents to clinic. Her parents have noted a squint for approximately 1
year. The orthoptic report is as follows:

Visual acuity: 6/9 right eye; 6/24 left eye


PCT (distance): 35 PD ET distance
PCT (near): 45 PD ET near
Refraction: +3.50D both eyes
The most appropriate initial management for this patient is:
 prescription of +3.50D in both eyes with add +3.50 in both eyes (bifocals)
 a Kestenbaum procedure (bilateral recess/resect procedure)
 prescription of +3.50D in both eyes and patching of the right eye
 bimedial recessions
 bilateral resections

 Your answer was CORRECT


 Explanation
 This may be a case of partially or totally accommodative (refractive) esotropia, which will
respond nicely to hyperopic correction. This child does have a clinically high AC/A ratio on
the basis that distance esotropia is 10PD greater than near. However, she should start with
standard hyperopic glasses. Bifocals might be added later if a significant residual esodeviation
remains at near with the distance correction. Her left amblyopia must of course be treated
aggressively with careful follow-up of the treatment effect on each eye

Which statement regarding motor fusion is FALSE?

a normal divergence amplitude is 14 PD at distance and 16 PD at near


motor fusion is a means of avoiding diplopia

motor fusion is the process by which similar retinal images are made to fall on corresponding
retinal areas

normal vertical fusional amplitude varies from 2 to 4 PD and is independent of fixation distance

a normal convergence amplitude is 14 PD at distance and 38 PD at near

Your answer was CORRECT


Explanation
Normal divergence amplitudes are 6 PD at distance and 16 PD at near.

Normal convergence amplitudes are 14PD at distance and 38PD at near.

Reference: Nelson LB, Olitsky S. Harley’s Pediatric Ophthalmology. Fifth edition. Philadelphia:
Lippincott Williams & Wilkins, 2005. pp 88
A 16-year-old student has an esotropia of 10 prism dioptres for distance and near. She takes the
Worth 4-dot test and reports seeing 4 lights.

What is the most appropriate conclusion?


she has left suppression
she has functional visual impairment
she has normal BSV
she has harmonious abnormal retinal correspondence
she has right suppression

Your answer was CORRECT


Explanation
A patient with a manifest squint who sees 4 lights with the Worth 4-dot test has harmonious
abnormal retinal correspondence (ARC).

In a Worth 4-dot test, there are 4 lights: 1 red, 2 green and 1 white.

The patient wears red-green glasses: green in front of right eye, red in front of left.

The following outcomes can be interpreted from the test:


 4 lights seen and patient is orthophoric = normal BSV
 4 lights seen and patient has manifest squint = harmonious ARC
 2 red lights seen = right suppression
 3 green lights seen = left suppression
 5 lights (2 red and 3 green) = diplopia

Note: if red glasses are worn in front of the right eye, and green in front of the left eye, then
suppression results are inverted: 2 red lights would mean left suppression while 3 green lights
would mean right suppression.

In the setting of a heterophoria, fusional vergence amplitudes may be diminished by all of the
following EXCEPT:
alcohol consumption
improvement in visual acuity
intercurrent illness
fatigue

Your answer was CORRECT


Explanation
Factors known to be associated with decompensated latent strabismus include:
 alcohol
 fatigue
 illness
 decreased visual acuity (e.g., cataract)
Which of the following findings on examination would be LEAST consistent with a diagnosis of
amblyopia:
prolonged reaction times to a stimulus
loss of 2 lines visual acuity with a neutral density filter
eccentric fixation
binocular suppression
decreased contrast sensitivity

Your answer was CORRECT


Explanation
A neutral density filter significantly reduces vision in organic disease (such as optic neuritis), but
generally does not in pure amblyopia.

Features of amblyopia:
 reduced Snellen acuity by 2 lines
 reduced visual acuity with crowded letters compared to single optotypes
 decreased contrast sensitivity
 binocular suppression
 eccentric fixation
 prolonged perception and reaction times to a stimulus
 no RAPD, no VF defect

Findings that favor the diagnosis of spasm of the near reflex rather than accommodative esotropia
include:

1. near myopia
2. esotropia worse at near than at distance
3. miosis on attempted lateral gaze
1 and 3
1, 2 and 3
3 only
2 only
1 only

Your answer was CORRECT


Explanation
Spasm of the near reflex can be distinguished from accommodative esotropia by:
 near myopia
 miosis on attempted lateral gaze
 no vertical component
 the angle of turn is highly variable and unpredictable

In which of the following types of strabismus is amblyopia LEAST likely?


partially-accommodative esotropia

alternating esotropia

infantile esotropia

esotropia associated with Duane's syndrome

exotropia associated with craniosynostosis

Your answer was CORRECT


Explanation
Amblyopia in Duane's syndrome is uncommon but is more likely than in alternating esotropia,
where each eye shares the visual workload.

Which is FALSE about the corneal light reflex tests?

the Krimsky test uses prisms in front of the fixing or non-fixing eye to center the deviated light
reflex

the Maddox rod can be used to help measure both cyclodeviations as well as horizontal or vertical
deviations

the corneal light reflex corresponds with the first Purkinje image

if strabismus is present, the fixing eye will have a brighter red reflex on Bruckner testing

Your answer was CORRECT


Explanation
The Bruckner test is a bilateral red reflex test, and if strabismus is present the brighter reflex is in
the deviated eye. This is because the light reflects from peripheral retina in the deviated eye; and
since there is less pigment in the peripheral retina than the macula, there is more reflection of light
from the peripheral retina of the deviated eye.

Note: in regards to the Krimsky test, the original test described by Krimsky involved placing
prisms over the deviated eye until the light reflex in the deviated eye was central. The convention
now (the modified Krimsky) involves placing the prism over the fixing eye, until the reflex in the
deviated eye is central; as it is easier to line up the light reflex when there is no prism over the eye
of interest (deviated eye). Thus it is correct to say that the Krimsky test involves the placement of
prisms over the fixing eye (modified Krimsky) or the non-fixing eye (original Krimsky).

An 18-month-old child undergoes bilateral medial rectus recessions for infantile esotropia. On the
first postoperative day, the deviation is measured as less than 10 prism diopters of residual
esotropia, with fairly good versions. At the 1-week visit, there is a prominent right exotropia,
which increases in left gaze. Duction testing demonstrates an inability to adduct the right eye past
the midline.
The most likely diagnosis is:

restriction of the right medial rectus due to orbital fat prolapse

lost or slipped muscle

iatrogenic Brown's syndrome

surgical overcorrection

surgical undercorrection

Your answer was CORRECT


Explanation
The findings suggest a lost or slipped right medial rectus muscle. Consecutive exotropia due to
overcorrection alone should not be associated with striking impairment in adduction.

Some practitioners do not recommend the full hyperopic correction in intermittent accommodative
esotropia because:

the greater deviation at near will not be fully compensated

distance vision will be blurred

the patient may become exotropic with full hyperopic correction

the patient may be converted to a constant esotropia without glasses

the problem never becomes constant

Your answer was CORRECT


Explanation
Correction with full hyperopic prescription may weaken the patient's fusional divergence, which is
the force keeping accommodative esotropia intermittent at its outset. Then, the esotropia may
become constant without the crutch of the spectacles.

A 5-year-old has a convergent squint. Visual acuities are 6/6 OD, 6/24 OS. Alternate cover test
shows left esotropia with and without distance glasses. Prism cover test shows 40 PD BO without
glasses for distance and 25PD BO with glasses for distance. Cycloplegic refraction is right
+4.00DS, left +6.00DS.

What is the most likely diagnosis?

sensory esotropia

fully accommodative esotropia


partially accommodative esotropia
near esotropia

Your answer was CORRECT


Explanation
The angle is reduced but not eliminated with spectacle correction of hypermetropia so this is
partially accommodative (as opposed to fully accommodative) esotropia.

You might also like